利用者:PSX/sandbox

提供: Mathpedia

[math] % preamble セクションでは任意のコードが非表示になります. % 多くの newcommand などを利用する場合は preamble 内で宣言してください.{{begin |preamble}} \lt !-- 任意のユーザー名に閲覧・編集を制限できます --\gt \lt !-- {{restrictPage |visibleTo=Administrator |editableBy=Administrator }} --\gt \lt math\gt % preamble セクションでは任意のコードが非表示になります. % 多くの newcommand などを利用する場合は preamble 内で宣言してください. \newcommand{\R}{\mathbb{R}} \newcommand{\C}{\mathbb{C}} \newcommand{\K}{\mathbb{K}} \newcommand{\Z}{\mathbb{Z}} \newcommand{\Q}{\mathbb{Q}} \newcommand{\Zp}{\mathbb{Z}_{\ge 1}} \newcommand{\Zz}{\mathbb{Z}_{\ge 0}} \newcommand{\M}{\mathbb{M}} \newcommand{\B}{\mathcal{B}} \newcommand{\D}{\mathcal{D}} \newcommand{\F}{\mathcal{F}} \newcommand{\G}{\mathcal{G}} \newcommand{\H}{\mathcal{H}} \newcommand{\I}{\mathcal{I}} \newcommand{\L}{\mathcal{L}} \newcommand{\n}{\mathbf{n}} \newcommand{\Rnp}{\mathbb{R}^n_+} \newcommand{\Rnn}{\mathbb{R}^n_-} \newcommand{\pRnp}{\mathbb{\partial R}^n_+} \newcommand{ae}{\mathrm{a.e.}\ } \newcommand{\inn}{\ \mathrm{in}\ } \newcommand{\on}{\ \mathrm{on}\ } \newcommand{\for}{\ \mathrm{for}\ } \newcommand{\jf}{\ \mathrm{if }\ } \newcommand{\det}{\operatorname{det}} \newcommand{\dim}{\operatorname{dim}} \newcommand{\Ker}{\operatorname{Ker}} \newcommand{\sgn}{\operatorname{sgn}} \newcommand{\wid}{\operatorname{wid}} \newcommand{\diam}{\operatorname{diam}} \newcommand{\dist}{\operatorname{dist}} \newcommand{\supp}{\operatorname{supp}} \newcommand{\avg}{\sout{\int}} \newcommand{\rcpt}{\subset\subset} \newcommand{\loc}{{\rm{loc}}} \newcommand{\implies}{\ \Rightarrow\ } \newcommand{\iff}{\ \Leftrightarrow\ } \newcommand{\norm}[1]{\lVert #1\rVert} \newcommand{\pardif}[2]{\frac{\partial{#1}}{\partial{#2}}} \newcommand{\pair}[1]{\langle #1\rangle} \newcommand{\Bb}{\overline{B}} \newcommand{\Ombar}{\overline{\Omega}} \newcommand{\OmT}{\Omega\cup T} \newcommand{\pOm}{{\partial\Omega}} \renewcommand{\Re}{\operatorname{Re}} \renewcommand{\Im}{\operatorname{Im}} \DeclareMathOperator*{\osc}{osc} \DeclareMathOperator*{\essosc}{ess\, osc} \DeclareMathOperator*{\esssup}{ess\, sup} \DeclareMathOperator*{\essinf}{ess\, inf} [/math]

以下の形の形式的な線型微分作用素 $L$ を考える:

  • $$Lu=-\sum_{i,j=1}^n a_{ij}(x)D_{ij}u+\sum_{i=1}^n b_i(x)D_iu+c(x)u. \tag{GF}\label{GF}$$
  • $$Lu=-\sum_{i=1}^n D_i\left(\sum_{j=1}^n a_{ij}(x)D_ju+b_i(x)u\right)+\sum_{i=1}^n c_i(x)D_iu+d(x)u. \tag{DF}\label{DF}$$

ここで $x\in\Omega$。

定義 1 (楕円性)

$L$ を(\ref{GF})あるいは(\ref{DF})の形の形式的な微分作用素とする。

  • (1) 各 $x\in\R^n$ について $\lambda(x)\gt 0$ であって

$$\sum_{i=1}^n a_{ij}(x)\xi_i\xi_j\ge\lambda(x)|\xi|^2\ (\forall x\in\Omega,\xi\in\R^n) \tag{E}\label{E}$$ をみたすものが存在するとき、$L$ は楕円型(elliptic)であるという。

  • (2) 定数 $\lambda\gt 0$ であって

$$\sum_{i=1}^n a_{ij}(x)\xi_i\xi_j\ge\lambda|\xi|^2\ (\forall x\in\Omega,\xi\in\R^n) \tag{SE}\label{SE}$$ をみたすものが存在するとき $L$ は狭義楕円型(strictly elliptic)であるという。

  • (3) 定数 $\Lambda\ge\lambda\gt 0$ であって

$$\lambda|\xi|^2\le\sum_{i,j}a_{ij}\xi_i\xi_j\le\Lambda|\xi|^2\ (\forall x\in\Omega,\xi\in\R^n)\tag{UE}\label{UE}$$ をみたすものが存在するとき $L$ は一様楕円型(uniformly elliptic)であるという。[1]

(\ref{GF})の形の楕円型作用素を非発散型(non-divergence form)または一般型(general form)の2階線型楕円型作用素といい、(\ref{DF})の形の楕円型作用素を発散型(divergence)の2階線型楕円型作用素という。

注意

作用する関数 $u$ について偏微分の順序交換 $D_{ij}u=D_{ji}u$ が成り立つ場合、非発散型の作用素 $L$ の係数関数 $a_{ij}$ を $\frac{1}{2}(a_{ij}+a_{ji})$ に置き換えることで $a_{ij}=a_{ji}$ を仮定することができる。$L$ が発散型の場合も $a_{ij}=a_{ji}$ を仮定することがある。

$a_{ij}=a_{ji}$ のとき、(\ref{E})は行列 $(a_{ij}(x))_{i,j}$ の固有値 $\mu_1(x)\le\ldots\le\mu_n(x)$ が $\lambda(x)\le\mu_1(x)\le\mu_n(x)\le\Lambda(x)$ をみたすことと同値である。

$a_{ij}$ が微分可能であるとき非発散型の作用素 $L$ を発散型 $$Lu=-\sum_{i=1}^n D_i\left(\sum_{j=1}^n a_{ij}D_ju\right)+\sum_{i=1}^n\left(b_i+\sum_{j=1}^n D_ja_{ij}\right)D_iu+cu$$ に書き直すことができる。逆に $a_{ij}$ と $b_i$ が微分可能であるとき発散型の作用素 $L$ を非発散型 $$Lu=-\sum_{i,j=1}^n a_{ij}D_{ij}u+\sum_{i=1}^n \left(-\sum_{j=1}^n D_ja_{ij}+b_i+c_i\right)D_iu+\left(\sum_{i=1}^n D_ib_i+d\right)u$$ に書き直すことができる。

以下では断りなく $\lambda$ と $\Lambda$ を(\ref{E})をみたすものを表す記号として用いる。

記法および注意

ここではとくに断らない限り実数値関数のみを取り扱う。また断りなく次の記法を用いる:

  • $n\in\Zp$ とし、$\Omega$ を $\R^n$ における開集合とする。
  • $x\in\R^n$、$r>0$ について $B_r(x)$ で $\R^n$ の半径が $r$、中心が $x$ の開球を表す。中心や半径を具体的に指定しない場合は $B_r$ や $B(x)$ などと略すことがある。
  • $\H^s$ で $s$ 次元Hausdorff測度、$\L^n$ で $n$ 次元Lebesgue外測度(Hausdorff測度とarea_coarea_formulaの定義30。命題31も参照。)を表し、$\L^n$ -a.e. を単にa.e.とかき、また誤解のおそれがなければ省略する。
  • 多重指数 $\alpha$ について $\alpha$ 階偏微分を $D^\alpha$ で表す。また $\alpha=ke_i$ のときは $D^\alpha u$ を $D^k_i u$ と表す。$D^1_i u$ は $D_iu$ と表し、$\R^n$ 値関数 $(D_1u,\ldots D_nu)$ を $Du$ と表す。
  • $n=1$ のときは $D^k_1 u$ を $u^{(k)}$ と表す。また $u^{(1)}$、$u^{(2)}$ はそれぞれ $u'$、$u^{\prime\prime}$ と表す。
  • $\mu\in\R^n\backslash\{0\}$ について、$D_\mu$ で $\mu$ -方向微分を表す。とくに $u$ が微分可能であるとき $D_\mu u=\mu\cdot Du$ である。
  • $p\in[1,\infty]$ について $p'$ でそのHölder共役指数、すなわち $p^{-1}+p'^{-1}=1$ をみたす $p'\in[1,\infty]$ を表す。
  • $\norm{\cdot}_{p;\Omega}$ で $L^p(\Omega)=L^p(\Omega,\L^n)$ のノルムを表す。また、相対開集合 $T\subset\pOm$ について $\norm{\cdot}_{p;T}$ で $L^p(T,\H^{n-1})$ のノルムを表す。誤解のおそれがなければ $\Omega$ と $T$ は省略することがある。
  • $C^k(\Ombar)$、$C^{k,\alpha}(\Ombar)$ のノルムにはHölder空間の基本事項で定義した記号を用いる。また、$W^{k,p}(\Omega)$ のノルムにはSobolev空間とSobolevの不等式で定義した記号を用いる。
  • $Q\colon=(-1,1)^n$、$Q_+\colon=(-1,1)^{n-1}\times(0,1)$、$Q_0\colon=(-1,1)^{n-1}\times\{0\}$ とする。
  • $\Omega,\Omega'\subset\R^n$ を開集合とし、$f\colon\Omega'-\Omega\to\R$ と $g\colon\Omega\to\R$ を $\L^n$ -可測関数とする。

$$f*g(x)\colon=\int_{\Omega}f(x-y)g(y)dy$$ が a.e. $x\in\Omega'$ で定義されるとき $f*g$ は $\Omega'$ 上で定義されるといい、これを $f$ と $g$ の合成積という。[2]

  • 実数値関数からなる線形空間 $X$ に対し、各成分が $X$ に属する $\R^m$ 値関数の空間を $m$ 個の $X$ の直積と同一視して $X^m$ とかく。

Laplace方程式とPoisson方程式

Poisson方程式とは方程式 $$-\Delta u=f\ \inn \Omega$$ を指す。ここで $f\colon\Omega\to\R$ は既知の関数であり、$\Delta$ はラプラシアン $\displaystyle\Delta u\colon=\sum_{i=1}^n D_i^2u$ である。とくに $f\equiv 0$ とした方程式 $$-\Delta u=0\ \inn \Omega$$ をLaplace方程式という。

明らかにラプラシアンは一様楕円型である。Laplace方程式およびPoisson方程式は2階線型楕円型方程式のもっとも単純な形であり、それらのモデルとして重要である。

定義 2 (劣(優)解、劣(優)調和関数)

$f\in C(\Omega)$ とする。各 $x\in\Omega$ について $-\Delta u(x)\le(\ge) f(x)$ をみたす $u\in C^2(\Omega)$ を Poisson方程式 $-\Delta u=f$ in $\Omega$ の劣(優)解という。Laplace方程式の(劣、優)解 $u\in C^2(\Omega)$ を(劣、優)調和関数という。

注意

$u$ を $-\Delta u=f$ in $\Omega$ の優解とすると、$-u$ は $-\Delta u=-f$ in $\Omega$ の劣解である。このことから、劣解 $u$ に関する命題において、その証明の $u$ を $-u$ に置き換えることによって優解 $u$ に関する同様の命題を得ることができる。以下では劣解および優解に関する多くの命題において $u$ が劣解の場合のみを示す。また、$u$ を方程式 $-\Delta u=f$ in $\Omega$ の解とすると $u$ は劣解かつ優解であるから、劣解および優解に関する命題から解に関する命題も直ちに得られる。こうした命題の証明も省略する。

(劣・優)調和関数の基本的な性質

命題 3 (平均値不等式)

$B=B_R(x_0)$ を球とする。$u\in C^2(B)\cap C(\overline{B})$ を $B$ 上の劣(優)調和関数とすると \begin{align*} u(x_0)&\le(\ge)\frac{1}{n\omega_nR^{n-1}}\int_{\partial B}ud\H^{n-1},\\ u(x_0)&\le(\ge)\frac{1}{\omega_nR^n}\int_B u. \end{align*} とくに $u$ が $B$ 上の調和関数であるときこれらの等号が成り立つ。

Proof.

$u$ が劣調和関数の場合のみを示す。$r\in[0,R]$ について $$\varphi(r)\colon=\begin{cases} \frac{1}{n\omega_nr^{n-1}}\int_{\partial B_r(x_0)}ud\H^{n-1}&\jf r\in(0,R]\\ u(x_0)&\jf r=0 \end{cases}$$ と定める。変数変換 $x=x_0+rz$ により $\varphi(r)$ は $$\varphi(r)=\int_{\partial B_1(0)}u(x_0+rz)d\H^{n-1}(z)$$ と表される。これより $\varphi$ は $[0,R]$ において連続である。また $\varphi$ は $(0,R)$ において微分可能で \begin{align*} \varphi'(r)&=\int_{\partial B_1(0)}Du(x_0+rz)\cdot zd\H^{n-1}(z)\\ &=\frac{1}{n\omega_nr^{n-1}}\int_{\partial B_r(x_0)}Du(x)\cdot\frac{x-x_0}{r}d\H^{n-1}\\ &=\frac{1}{n\omega_nr^{n-1}}\int_{B_r(x_0)}\Delta u\\ &\ge 0. \end{align*} ここで $x\in\partial B_r(x_0)$ における $\partial B_r(x_0)$ の外向き単位法線ベクトルが $\displaystyle \frac{x-x_0}{r}$ と表されることを用いた。

従って $\varphi$ は $[0,R]$ で単調増加であり任意の $r\in[0,R]$ について $$u(x_0)=\varphi(0)\le\varphi(r)=\frac{1}{n\omega_nr^{n-1}}\int_{\partial B_r(x_0)}ud\H^{n-1}.$$ $r=R$ として1つ目の不等式を得る。また $\partial B_r(x_0)=\{x\in\R^n\colon|x-x_0|=r\}$ に注意してcoarea formulaを用いると $$u(x_0)=\frac{1}{\omega_nR^n}\int_0^R n\omega_nr^{n-1}u(x_0)dr\le\frac{1}{\omega_nR^n}\int_0^R\left(\int_{\partial B_r(x_0)}ud\H^{n-1}\right)dr=\frac{1}{\omega_nR^n}\int_B u.$$

注意

$u$ が調和関数の場合はとくに平均値定理や平均値性質と呼ばれることも多い。

命題 4 (平均値性質の逆)

逆に $u\in C(\Omega)$ が任意の球 $B=B_R(x_0)\rcpt\Omega$ について $\displaystyle \frac{1}{n\omega_nR^{n-1}}\int_{\partial B}ud\H^{n-1}=u(x_0)$ をみたすとき $u\in C^\infty(\Omega)$ であって $u$ は調和関数となる。

Proof.

$\eta_\varepsilon=\eta_\varepsilon(|x|)$ を球対称なFriedrich's mollifier(Sobolev空間とSobolevの不等式の定義11)とする。$\Omega'\rcpt\Omega$ として $\varepsilon\gt 0$ を十分小さくとると $x\in\Omega$ について $$\eta_\varepsilon*u(x)=\int_{B_\varepsilon(x)}\eta_\varepsilon(x-y)u(y)dy=\int_0^\varepsilon\left(\int_{\partial B_r(x)}ud\H^{n-1}\right)\eta_\varepsilon(r)dr=\int_0^r n\omega_nr^{n-1}u(x)\eta_\varepsilon(r)dr=u(x)\int_{B_\varepsilon(0)}\eta_\varepsilon=u(x).$$ よって $u=\eta_\varepsilon*u\in C^\infty(\overline{\Omega'})$ となり $u\in C^\infty(\Omega)$。

また命題 3の証明と同様の計算により $x\in\Omega$ と十分小さい $r\gt 0$ について $$\int_{B_r(x)}\Delta u=\pardif{}{r}\frac{1}{n\omega_nr^{n-1}}\int_{\partial B_r(x_0)}ud\H^{n-1}=0.$$ これより $-\Delta u(x)=0$ を得る。

系 5

$u\in C^2(\Omega)$ を調和関数とすると $u\in C^\infty(\Omega)$。

系 6

$\{u_m\}_{m=1}^\infty\subset C^2(\Omega)$ は調和関数の列で、ある $u\in C(\Omega)$ に広義一様収束しているとする。このとき $u$ も調和関数である。

Proof.

任意の球 $B=B_R(x_0)\rcpt\Omega$ について $$u(x_0)=\lim_{m\to\infty}u_m(x_0)=\lim_{m\to\infty}\frac{1}{n\omega_nR^{n-1}}\int_{\partial B}u_md\H^{n-1}=\frac{1}{n\omega_nR^{n-1}}\int_{\partial B}ud\H^{n-1}.$$

定理 7 (強最大値原理)

$\Omega$ は連結であるとする。$u\in C^2(\Omega)$ を $\Omega$ 上の劣(優)調和関数とすると $u$ は次のいずれかをみたす:

  • (i) $u$ は定数関数である。
  • (ii) $u$ は $\Omega$ において最大(小)値をとらない。
Proof.

$u$ が劣調和関数の場合のみを示す。

$u$ が $\Omega$ において最大値 $M$ をとったとし、$\Omega_M\colon=\{x\in\Omega\colon u(x)=M\}$ とする。明らかに $\Omega_M$ は $\Omega$ における相対位相について閉集合である。一方、$x_0\in\Omega_M$ とし、$B=B_r(x_0)\subset\Omega$ とすると平均値不等式より $$\frac{1}{\omega_nR^n}\int_B u\ge M=\frac{1}{\omega_nR^n}\int_B M.$$ $u\le M$ in $B$ であるから $u=M\ \inn B$ となる。これより $B\subset\Omega_M$ であり、$\Omega_M$ は開集合である。

定義より明らかに $\Omega_M\neq\emptyset$ で、$\Omega$ は連結であったから $\Omega_M=\Omega$。すなわち $u\equiv M\ \inn\Omega$。

定義 8 (上境界値、下境界値)

$u\colon\Omega\to\R$ に対し $u^*,u_*\colon\Ombar\to\R$ を $$u^*(x)\colon=\begin{cases} u(x)&\jf x\in\Omega\\ \limsup_{y\in\Omega,y\to x}u(y) &\jf x\in\pOm \end{cases}, u_*(\xi)\colon=\begin{cases} u(x)&\jf x\in\Omega\\ \liminf_{y\in\Omega,y\to x}u(y) &\jf x\in\pOm \end{cases}$$ と定め、$u^*|_\pOm$、$u_*|_\pOm$ をそれぞれ $u$ の上境界値、下境界値という。

$u\in C(\Ombar)$ のときは $u^*=u_*=u$ on $\Ombar$ が成り立つ。

命題 9

$u\colon\Omega\to\R$ とすると $\xi\in\pOm$ について $$\limsup_{x\in\Ombar,x\to\xi}u^*(x)=u^*(\xi),\liminf_{x\in\Ombar,x\to\xi}u_*(x)=u_*(\xi).$$ とくに $u$ が上半連続のとき $u^*$ も上半連続で、$u$ が下半連続のとき $u_*$ も下半連続である。

Proof.

$u^*$ についてのみ示す。$\displaystyle \limsup_{\substack{x\in\Ombar,\\x\to\xi}}u^*(x)\ge u^*(\xi)$ は $u^*$ の定義より直ちに従う。

$\displaystyle \limsup_{\substack{x\in\Ombar,\\x\to\xi}}u^*(x)\le u^*(\xi)$ を示す。$x_m\to\xi\ (m\to\infty)$ をみたす任意の $\{x_m\}_{m=1}^\infty\subset\Ombar$ が $\displaystyle \limsup_{m\to\infty}u^*(x_m)\le u^*(\xi)$ をみたすことを示せばよい。

$x_m'\in\Omega$ を次のようにとる: $x_m\in\Omega$ のときは $x_m'=x_m$ とする。$x_m\in\pOm$ のときは $u^*$ の定義より $x_m'\in\Omega$ を $|x_m'-x_m|\lt 2^{-m}$ かつ $u^*(x_m)\lt u(x_m')+2^{-m}$ となるようにとれる。

このとき $|x_m'-\xi|\le|x_m'-x_m|+|x_m-\xi|\to 0\ (m\to\infty)$ より $\displaystyle\limsup_{m\to\infty} u(x_m')\le u^*(\xi)$ であるから $$\limsup_{m\to\infty}u^*(x_m)\le\limsup_{m\to\infty}(u(x_m')+2^{-m})\le u^*(\xi).$$

定理 10 (弱最大値原理)

$\Omega$ は有界であるとする。$u\in C^2(\Omega)$ を $\Omega$ 上の劣(優)調和関数とすると $$\sup_\Omega u\left(\inf_\Omega u\right)=\sup_\pOm u^*\left(\inf_\pOm u_*\right).$$ とくに $u\in C^2(\Omega)\cap C(\Ombar)$ のとき $$\sup_\Omega u\left(\inf_\Omega u\right)=\sup_\pOm u\left(\inf_{\pOm}u\right).$$

Proof.

$u$ が劣調和関数の場合のみを示す。

$\displaystyle \sup_\Omega u\ge\sup_\pOm u^*$ は明らか。

$\displaystyle \sup_\Omega u\gt\sup_\pOm u^*$ であったとする。$u^*$ は上半連続で $\Ombar$ はコンパクトであるから $u^*$ はある $x_0\in\Ombar$ において最大値をとる。このとき $\displaystyle\sup_\pOm u^*\lt\sup_\Omega u=\max_\Ombar u^*=u^*(x_0)$ となるので $x_0\in\Omega$。よって $u$ は $\Omega$ において最大値 $u(x_0)$ をとり、強最大値原理より $u$ は $x_0$ の属する連結成分 $\Omega_0$ 上で定数である。とくに $u=u(x_0)$ on $\pOm_0$ となるが、これは仮定に矛盾する。

注意

$u$ が優調和関数の場合を最小値原理という場合も多い。

Laplace方程式においては最大値原理は平均値不等式から直ちに得られる系であるが、後の節で述べるようにより一般の2階楕円型方程式についても成り立つという点において平均値不等式より重要である。

系 11 (比較原理)

$\Omega$ は有界であるとする。$u,v\in C^2(\Omega)$ が $-\Delta u\le-\Delta v$ in $\Omega$ かつ $(u-v)^*\le 0$ on $\pOm$ をみたすとすると $u\le v$ in $\Omega$。また $\Omega$ が連結であるすると $u\not\equiv v$ でなければ $u\lt v$ in $\Omega$。

Proof.

$w\colon=u-v$ とすると $-\Delta w\le 0$ in $\Omega$ かつ $w\le 0$ on $\pOm$ となるので $w\le 0$ in $\Omega$。後半も $w$ に強最大値原理を用いれば従う。

参考

比較原理も同様に多くの2階楕円型方程式がもつ性質である。比較原理は次の意味で「非常に強力」である:方程式の解 $u$ に対する評価を境界上で $v\ge(\le)u$ となる優(劣)解 $v$ を見つけるだけで作ることができる。この方法は $v$ を見つけることができれば具体的な計算だけで証明が完成し、複雑な理論を必要としない。それにもかかわらず、しばしば解に対して他の方法では得ることが難しい強力な評価や可解性の結果を与える。

系 12 (Poisson方程式のDirichlet問題の古典解の一意性)

$\Omega$ は有界であるとする。$u,v\in C^2(\Omega)\cap C(\Ombar)$ が $-\Delta u=-\Delta v$ in $\Omega$ かつ $u=v$ on $\pOm$ をみたすとすると $u=v$ in $\Omega$。

定理 13 (Harnackの不等式)

$\Omega$ は連結であるとする。$u\in C^2(\Omega)$ を非負値の調和関数とすると任意の連結部分領域 $\Omega'\rcpt\Omega$ について $$\sup_{\Omega'}u\le C_{\Omega,\Omega'}\inf_{\Omega'}u.$$

Proof.

$B=B_r(x_0)$ が $B_{3r}(x_0)\rcpt\Omega$ をみたすとする。平均値性質により $x,y\in B$ について $$u(x)=\frac{1}{\omega_n(2r)^n}\int_{B_{2r}(x)}u\le\frac{1}{\omega_n(2r)^n}\int_{B_{3r}(x_0)} u=\left(\frac{3}{2}\right)^nu(x_0)=\frac{\left(\frac{3}{2}\right)^n}{\omega_nr^n}\int_{B_r(x_0)}u\le\frac{\left(\frac{3}{2}\right)^n}{\omega_nr^n}\int_{B_{2r}(y)}u=3^nu(y).$$ $r\colon=\frac{d}{4}$ とする。$\Omega'\rcpt\Omega$ より $x_1,\ldots,x_N\in\Omega'$ を $\displaystyle \Omega'\subset\bigcup_{i=1}^N B_r(x_i)$ となるようにとれる。$x,y\in\Omega'$ とするとHölder空間の基本事項の補題38より $y_0,\ldots,y_m\in\Omega'$、$i_0,\ldots,i_{m-1}\in\{1,\ldots,N\}$、$m\le N$ であって $y_0=x$、$y_m=y$ かつ $y_j,y_{j+1}\in B_r(x_{i_j})$ をみたすものが存在し、各 $j$ について $B_{3r}(x_{i_j})\rcpt\Omega$ より $u(y_j)\le 3^nu(y_{j+1})$ が成り立つ。これより $$u(x)=u(y_0)\le 3^nu(y_1)\le\ldots\le 3^{mn}u(y_m)=3^{mn}u(y).$$ 従って $$\sup_{\Omega'}u\le 3^{Nn}\inf_{\Omega'}u.$$

基本解とNewtonian potential

命題 14 (Laplace方程式の球対称解)
  • (1) $A\colon=\{x\in\R^n\colon a\lt|x|\lt b\}$、$0\le a\lt b\le\infty$ を円環領域とする。球対称関数 $u(x)=u(|x|)\in C^2(A)$ について

$$\Delta u(x)=u^{\prime\prime}(|x|)+\frac{n-1}{r}u'(|x|)\for x\in A.$$

  • (2) Laplace方程式 $-\Delta u=0$ in $\R^n\backslash\{0\}$ の球対称解 $u(x)=u(|x|)\in C^2(\R^n\backslash\{0\})$ は定数 $c_1,c_2\in\R$ を用いて

$$u(x)=\begin{cases} c_1|x|^{2-n}+c_2&\jf n\ge 3\\ -c_1\log|x|+c_2&\jf n=2 \end{cases}$$ と表される。

Proof.

(1)を示す。$i=1,\ldots,n$ について $$D_iu(x)=\frac{x_i}{r}u'(r),D_{ii}u(x)=\frac{x_i^2}{r^2}u^{\prime\prime}(r)+\left(\frac{1}{r}-\frac{x_i}{r^3}\right)u'(r)$$ となるので $$\Delta u(x)=\sum_{i=1}^n\left(\frac{x_i^2}{r^2}u^{\prime\prime}(r)+\left(\frac{1}{r}-\frac{x_i}{r^3}\right)u'(r)\right)=u^{\prime\prime}(r)+\frac{n-1}{r}u'(r).$$

(2)を示す。(1)より常微分方程式 $$u^{\prime\prime}(r)+\frac{n-1}{r}u'(r)=0\ (r\in(0,\infty))$$ を解けばよい。変数分離法により $u'(r)$ は定数 $c$ を用いて $u'(r)=cr^{1-n}$ と表され、これを積分すれば主張が従う。

定義 15 (Laplace方程式の基本解)

$\Gamma\colon\R^n\backslash\{0\}\to\R$ を $$\Gamma(x)=\Gamma(|x|)\colon=\begin{cases} -\frac{1}{2\pi}\log|x|&\jf n=2\\ \frac{1}{n(n-2)\omega_n}|x|^{2-n}&\jf n\ge 3 \end{cases}\tag{N}\label{dNP}$$ により定め、Laplace方程式の基本解という。$\Gamma$ は $-\Delta\Gamma=0$ in $\R^n\backslash\{0\}$ の球対称解のひとつである。

命題 16 ($\Gamma$ の評価)

$\Gamma$ は $\R^n\backslash\{0\}$ において任意回微分可能で次をみたす:

  • (1)多重指数 $\alpha\neq 0$ について

$$|D^\alpha\Gamma(x)|\le C_{n,\alpha}|x|^{2-n-|\alpha|}\for x\in\R^n\backslash\{0\}.$$

  • (2) $r\in(0,1)$ について

$$\int_{B_r(0)}\Gamma\le\begin{cases} Cr^2(-\log r+1)&\jf n=2\\ C_nr^2&\jf n\ge 3 \end{cases}.$$ とくに $\alpha\lt 2$ について $$r^{-\alpha}\int_{B_r(0)}\Gamma\to 0\ (r\to +0).$$

  • (3) $r\in(0,1)$ について

$$\int_{B_r(0)}|D\Gamma|\le C_nr.$$

Proof.

$x=(x_1,\ldots,x_n)\in\R^n\backslash\{0\}$ について $D_i\Gamma(x)=-\frac{1}{n\omega_n}|x|^{-n}x_i$ に注意すると $|\alpha|\ge 1$ に関する帰納法により多重指数 $\alpha\neq 0$ について $D^\alpha\Gamma$ は $|\alpha|$ 次斉次多項式 $P_\alpha$ を用いて $D^\alpha\Gamma(x)=|x|^{2-n-2|\alpha|}P_\alpha(x)$ と表される。これより $|D^\alpha\Gamma(x)|\le C|x|^{2-n-|\alpha|}$ を得る。また $r\in(0,1)$ について $$\int_{B_r(0)}\Gamma\le C\int_0^r \rho^{n-1}\Gamma(\rho)d\rho\le\begin{cases} C\int_0^r \rho(-\log\rho)d\rho\le C\rho^2(-\log\rho+1)&\jf n=2\\ C\int_0^r \rho d\rho\le Cr^2&\jf n\ge 3 \end{cases}$$ で $$\int_{B_r(0)}|D\Gamma|\le C\int_{B_r(0)}|x|^{1-n}dx\le C\int_0^rd\rho=Cr.$$

定義 17 (Newtonian potential)

$f\in L^1_\loc(\Omega)$ とする。合成積 $\Gamma*f$ が $\Omega$ 上で定義されるときこれを $f$ のNewtonian potentialという。

$n=2$ の場合は任意の $q\in[1,\infty)$ について $\Gamma\in L^q_\loc(\R^2)$ であるから $\Omega$ が有界であって $f\in L^1(\Omega)$ であれば $\Gamma*f$ は定義される。$n\ge 3$ の場合はSobolev空間とSobolevの不等式の命題57より $|\Omega|\lt\infty$ であって $f\in L^1(\Omega)$ であれば $\Gamma*f$ が定義される。

Newtonian potentialは次の意味で「 $-\Delta u=f$ の解 $u$ の候補」を与えるものである:

命題 18 (Greenの表現公式)

$\Omega$ を $C^{0,1}$ 領域(Hölder空間の基本事項の定義28。Sobolev空間とSobolevの不等式の定理90も参照。)とする。$\supp u$ が有界な $u\in C^2(\Ombar)$ について $$u(x)=\int_\Omega \Gamma(x-y)(-\Delta u(y))dy+\int_\pOm (\Gamma(x-y)Du(y)\cdot\nu(y)+(D\Gamma(x-y)\cdot\nu(y))u(y))d\H^{n-1}(y).$$ ここで $\nu$ は $\pOm$ の外向き単位法線ベクトルである。

とくに $u\in C^2_c(\R^n)$ について $$u=\Gamma*(-\Delta u)\ \inn\ \R^n.$$

Proof.

$x\in\Omega$ とし、$r\gt 0$ を十分小さくとって $B_r(x)\rcpt\Omega$ となるようにする。部分積分により \begin{align*} &\int_{\Omega\backslash B_r(x)} \Gamma(x-y)(-\Delta u(y))dy\\ &=-\int_\pOm \Gamma(x-y)Du(y)\cdot\nu(y)d\H^{n-1}(y)+\int_{\partial B_r(x)}\Gamma(x-y)Du(y)\cdot\frac{y-x}{r}d\H^{n-1}(y)-\int_{\Omega\backslash B_r(x)} D\Gamma(x-y)\cdot Du(y)dy\\ &=-\int_\pOm (\Gamma(x-y)Du(y)\cdot\nu(y)+(D\Gamma(x-y)\cdot\nu(y))u(y))d\H^{n-1}(y)\\ &\qquad+\int_{\partial B_r(x)}(\Gamma(x-y)Du(y)+D\Gamma(x-y)u(y))\cdot\frac{y-x}{r}d\H^{n-1}(y). \end{align*} ここで $$\left|\int_{\partial B_r(x)}\Gamma(x-y)Du(y)\cdot\frac{y-x}{r}d\H^{n-1}(y)\right|\le\Gamma(r)\cdot n\omega_nr^{n-1}\to 0\ (r\to +0)$$ で、 $$\int_{\partial B_r(x)}D\Gamma(x-y)u(y)\cdot\frac{y-x}{r}d\H^{n-1}(y)=-\Gamma'(r)\int_{\partial B_r(x)}ud\H^{n-1}=\frac{1}{n\omega_nr^{n-1}}\int_{\partial B_r(x)}ud\H^{n-1}\to u(x)\ (r\to +0)$$ であるから $r\to +0$ とすれば $$\int_\Omega \Gamma(x-y)(-\Delta u(y))dy=-\int_\pOm (\Gamma(x-y)Du(y)\cdot\nu(y)+(D\Gamma(x-y)\cdot\nu(y))u(y))d\H^{n-1}(y)+u(x)$$ を得る。

$-\Delta\Gamma=0$ より境界積分の項は調和関数になるので、$f=-\Delta u$ とすると $w=\Gamma*f$ も $w\in C^2(\Omega)$ であって $-\Delta w=f$ の解になっていることがわかる。また $u$ が調和関数の場合 $$u(x)=\int_\pOm (\Gamma(x-y)Du(y)\cdot\nu(y)+(D\Gamma(x-y)\cdot\nu(y))u(y))d\H^{n-1}(y)$$ という表示を得る。この表示からも系 5が成り立つことを確認できる。

一方、$f\in C(\Ombar)$ であっても $\Gamma*f\notin C^2(\Omega)$ となり、$u=\Gamma*f$ が $-\Delta u=f$ in $\Omega$ の解にならない場合がある。

$B=B_2(0)$ とし、$\eta\in C^2_c(B)$ を $\eta=1$ in $B_1(0)$ となるようにとる。$P(x)\colon=x_1x_2$ とすると $-\Delta(\eta P)=0$ in $B_1(0)$ となる。次のように $$f(x)\colon=\begin{cases} -\frac{1}{k}\Delta(\eta P)(2^kx)&\jf 2^{-k}\le|x|\lt 2^{1-k},k\in\Zz\\ 0&\jf x=0 \end{cases}$$ とおく。明らかに $\supp f$ はコンパクトである。また $|x|=2^{-k},k\in\Zz$ とすると $-\frac{1}{k}\Delta(\eta P)(2^kx)=-\frac{1}{k+1}\Delta(\eta P)(2^{k+1}x)=0$ であるから $f$ は $B\backslash\{0\}$ において連続であり、$0\lt|x|\lt 2^{-k},k\in\Zz$ とすると $|f(x)|\le \frac{1}{k}|-\Delta(\eta P)|_0$ となるので $|f(x)|\to 0\ (|x|\to +0)$ も成り立ち、$f$ は $B$ 上で連続である。

$\Gamma*f$ を求める。$l\in\Zp$ について $$f_l(x)\colon=\begin{cases} -\frac{1}{k}\Delta(\eta P)(2^kx)&\jf 2^{-k}\le|x|\lt 2^{1-k},k\in\{0,\ldots,l\}\\ 0&\jf |x|\lt 2^{-l} \end{cases} ,\quad w_l(x)\colon=\sum_{k=0}^l\frac{1}{2^{2k}k}(\eta P)(2^kx)$$ とすると $f_l\in C_c(B)$、$w_l\in C^2_c(B)$、$-\Delta w_l=f_l$ であるから $\Gamma*f_l=w_l$。また $l\to\infty$ とすると $f_l$ は $f$ に一様収束し、$w_l$ は $$w(x)\colon=\sum_{k=0}^\infty\frac{1}{2^{2k}k}(\eta P)(2^kx)$$ により定まる $w$ に 一様収束する。これより $\Gamma*f=w$。

$w$ を定める級数は $x\neq 0$ のときは有限項を除いて $0$ であることに注意すると $$w\in C^2(B\backslash\{0\}),\quad D^\alpha w(x)=\sum_{k=0}^\infty\frac{1}{k}D^\alpha(\eta P)(2^kx)\for x\in B\backslash\{0\},|\alpha|\le 2.$$ 一方 $D_{12}P=1$ に注意すると $2^{-l}\le|x|\lt 2^{1-l},l\in\Zp$ について $$D_{12}w(x)=\sum_{k=0}^l D_{12}(\eta P)(2^kx)=\sum_{k=0}^{l-1} D_{12}P(2^kx)+D_{12}(\eta P)(2^lx)\ge l-|D_{12}(\eta P)|_0.$$ これより $D_{12}w(x)\to +\infty\ (|x|\to +0)$ となり、$D_{12}w\notin C(B)$。これより $w=\Gamma*f\notin C^2(B)$ である。

一方で、以下に示すように、$f$ がHölder連続であれば $\Gamma*f\in C^2(\Omega)$、$-\Delta(\Gamma*f)=f$ in $\Omega$ が成り立つ。(上で示した例は $0$ においてHölder連続となっていないことに注意せよ。)

補題 19

$\Omega$ は有界であるとする。$f\in L^\infty(\Omega)$ とすると $\Gamma*f$ は $\R^n$ 上で定義され $$\Gamma*f\in C^1(\R^n),\quad D_i(\Gamma*f)=D_i\Gamma*f\ \inn \R^n.$$

Proof.

$i\in\{1,\ldots,n\}$ とする。$\zeta\in C^\infty(\R^n)$ を $0\le\zeta\le 1$ in $\R^n$、$\zeta=0$ in $B_1(0)$ かつ $\zeta=1$ on $\R^n\backslash B_2(0)$ となるようにとり、$\varepsilon\gt 0$ について $\zeta_\varepsilon(x)\colon=\zeta(\varepsilon^{-1}x)$ とする。$\zeta_\varepsilon$ は $$0\le\zeta_\varepsilon\le 1 \inn \R^n,\quad \zeta_\varepsilon=0\ \inn B_\varepsilon(0),\quad\zeta_\varepsilon=1\ \on \R^n\backslash B_{2\varepsilon}(0)$$ をみたす。

このとき $\zeta_\varepsilon\Gamma\in C^\infty(\R^n)$ となるので $$(\zeta_\varepsilon\Gamma)*f\in C^\infty(\R^n),D_i((\zeta_\varepsilon\Gamma)*f)=(D_i(\zeta_\varepsilon\Gamma))*f.$$ また $x\in\R^n$ について $$|(\zeta_\varepsilon\Gamma)*f(x)-\Gamma*f(x)|\le\int_\Omega (1-\zeta_\varepsilon(x-y))|\Gamma(x-y)||f(y)|dy\le\int_{B_{2\varepsilon}(x)}|\Gamma(x-y)|\norm{f}_\infty dy,$$ \begin{align*} |(D_i(\zeta_\varepsilon\Gamma))*f(x)-D_i\Gamma*f(x)|&\le\int_\Omega (1-\zeta_\varepsilon(x-y))|D_i\Gamma(x-y)||f(y)|dy+\varepsilon^{-1}\int_\Omega |D_i\zeta(\varepsilon^{-1}(x-y))||\Gamma(x-y)||f(y)|dy\\ &\le\int_{B_{2\varepsilon}(x)}|D_i\Gamma(x-y)|\norm{f}_\infty dy+\varepsilon^{-1}\norm{D_i\zeta}_\infty\int_{B_{2\varepsilon}(x)}|\Gamma(x-y)|\norm{f}_\infty dy \end{align*} であるから $$\sup_{\R^n}|(\zeta_\varepsilon\Gamma)*f-\Gamma*f|\le\norm{f}_\infty\int_{B_{2\varepsilon}(0)}|\Gamma|\to 0\ (\varepsilon\to +0),$$ $$\sup_{\R^n}|(D_i(\zeta_\varepsilon\Gamma))*f-D_i\Gamma*f|\le\norm{f}_\infty\int_{B_{2\varepsilon}(0)}|D_i\Gamma|+\varepsilon^{-1}\norm{D_i\zeta}_\infty\norm{f}_\infty\int_{B_{2\varepsilon}}|\Gamma|\to 0\ (\varepsilon\to +0).$$ これより主張が従う。

定理 20

$\Omega$ は有界であるとする。$f\in C^{0,\alpha}(\Omega)\cap L^\infty(\Omega)$、$\alpha\in(0,1)$ とすると $\Gamma*f\in C^2(\Omega)$。また $\Omega_0$ を $\Omega$ を含む $C^{0,1}$ 領域とすると $$D_{ij}(\Gamma*f)(x)=\int_\Omega D_{ij}\Gamma(x-y)(f(y)-f(x))dy-f(x)\left(\int_{\Omega_0\backslash\Omega}D_{ij}\Gamma(x-y)dy+\int_{\pOm_0}D_i\Gamma(x-y)\nu_j(y)d\H^{n-1}(y)\right)\ \for\ x\in\Omega.$$ ここで $\nu=(\nu_1,\ldots,\nu_n)$ は $\pOm_0$ の外向き単位法線ベクトルである。また $$-\Delta(\Gamma*f)=f\ \inn \Omega.$$

Proof.

$i,j\in\{1,\ldots,n\}$ とする。補題 19と同様に $\zeta_\varepsilon$ をとる。補題 19と同様の評価により $$\sup_\Omega|(\zeta_\varepsilon D_i\Gamma)*f-D_i\Gamma*f|\to 0\ (\varepsilon\to +0).$$ $x\in\Omega$ とする。$x\in\Omega'\rcpt\Omega$ となる開集合 $\Omega'$ をとり、$\varepsilon\in(0,\frac{1}{2}\dist(\Omega',\pOm))$ とする。$B_{2\varepsilon}(x)\cap(\Omega_0\backslash\Omega)$ より $y\in\Omega_0\backslash\Omega$ については $\zeta_\varepsilon(y)=1$ となることに注意するとGaussの発散定理により \begin{align*} \int_{\pOm_0}D_i\Gamma(x-y)d\H^{n-1}(y)&=\int_{\pOm_0}\zeta_\varepsilon(x-y)D_i\Gamma(x-y)d\H^{n-1}(y)\\ &=-\int_{\Omega_0}D_j(\zeta_\varepsilon D_i\Gamma)(x-y)dy\\ &=-\left(\int_\Omega D_j(\zeta_\varepsilon D_i\Gamma)(x-y)dy+\int_{\Omega_0\backslash\Omega}D_{ij}\Gamma(x-y)dy\right).\tag{*}\label{np1} \end{align*} $D_i((\zeta_\varepsilon\Gamma)*f)=D_i(\zeta_\varepsilon\Gamma)*f$ と同様に $D_j((\zeta_\varepsilon D_i\Gamma)*f)=D_j(\zeta_\varepsilon D_i\Gamma)*f$ も成り立つので(\ref{np1})より \begin{align*} D_j((\zeta_\varepsilon D_i\Gamma)*f)(x)&=\int_\Omega D_j(\zeta_\varepsilon D_i\Gamma)(x-y)f(y)dy\\ &=\int_\Omega D_j(\zeta_\varepsilon D_i\Gamma)(x-y)(f(y)-f(x))dy\\ &\qquad-f(x)\left(\int_{\Omega_0\backslash\Omega}D_{ij}\Gamma(x-y)dy+\int_{\pOm_0}D_i\Gamma(x-y)\nu_j(y)d\H^{n-1}(y)\right). \end{align*} ここで \begin{align*} \int_\Omega |D_{ij}\Gamma(x-y)||f(y)-f(x)|dy&\le C\int_\Omega |x-y|^{-n}|f(y)-f(x)|dy\\ &\le C\left(\int_{B_{2\varepsilon}(x)}[f]_{0,\alpha;\Omega'}|x-y|^{\alpha-n}dy+\int_{\Omega\backslash B_{2\varepsilon}(x)}2|f|_{0;\Omega'}|x-y|^{-n}dy\right)\lt\infty \end{align*} より積分 $$\int_\Omega D_{ij}\Gamma(x-y)(f(y)-f(x))dy$$ は定義される。$\Omega$ 上の関数 $g$ を $$g(x)\colon=\int_\Omega D_{ij}\Gamma(x-y)(f(y)-f(x))dy-f(x)\left(\int_{\Omega_0\backslash\Omega}D_{ij}\Gamma(x-y)dy+\int_{\pOm_0}D_i\Gamma(x-y)\nu_j(y)d\H^{n-1}(y)\right)$$ により定めると \begin{align*} |D_j((\zeta_\varepsilon D_i\Gamma)*f)(x)-g(x)|&\le\int_\Omega |D_j(\zeta_\varepsilon D_i\Gamma)(x-y)-D_{ij}\Gamma(x-y)||f(y)-f(x)|dy\\ &\le\int_\Omega ((1-\zeta_\varepsilon(x-y))|D_{ij}\Gamma(x-y)|+|D_j\zeta_\varepsilon(x-y)||D_i\Gamma(x-y)|)|f(y)-f(x)|dy\\ &\le C\int_{B_{2\varepsilon}(x)} (|x-y|^{-n}+\varepsilon^{-1}|D_j\zeta|_0|x-y|^{1-n})[f]_{0,\alpha;\Omega'}|x-y|^\alpha dy\\ &\le C\int_{B_{2\varepsilon}(x)}(|x-y|^{\alpha-n}+\varepsilon^{-1}|x-y|^{\alpha+1-n})[f]_{0,\alpha;\Omega'}dy\\ &\le C\varepsilon^{\alpha}[f]_{0,\alpha;\Omega'}. \end{align*} 従って $$\sup_{\Omega'}|D_j((\zeta_\varepsilon D_i\Gamma)*f)-g|\to 0\ (\varepsilon\to +0).$$ 以上より $D_i(\Gamma*f)=D_i\Gamma*f\in C^1(\Omega)$ となり、 $$\Gamma*f\in C^2(\Omega),\quad D_{ij}(\Gamma*f)=g\inn \Omega.$$ $\Delta(\Gamma*f)=f$ を示す。$x\in\Omega$ として $\Omega_0=B_R(x)$ を $\Omega$ を含む球とする。$-\Delta\Gamma=0$ in $\R^n\backslash\{0\}$ に注意すると \begin{align*} -\Delta(\Gamma*f)(x)&=\sum_{i=1}^n\left(-\int_\Omega D_{ii}\Gamma(x-y)(f(y)-f(x))dy\\ &\qquad+f(x)\left(\int_{\Omega_0\backslash\Omega}D_{ii}\Gamma(x-y)dy-\int_{\partial B_R(x)}D_i\Gamma(x-y)\frac{y_i-x_i}{R}d\H^{n-1}(y)\right)\right)\\ &=f(x)\int_{\partial B_R(x)}(-D\Gamma(x-y))\cdot\frac{x-y}{R}d\H^{n-1}\\ &=f(x)\int_{\partial B_R(x)}\frac{1}{n\omega_n}|x-y|^{-n}(x-y)\cdot\frac{x-y}{R}d\H^{n-1}\\ &=f(x)\cdot n\omega_nR^{n-1}\cdot \frac{1}{n\omega_n}R^{1-n}\\ &=f(x). \end{align*}

Green関数

Greenの表現公式は次の意味で「不十分」である: Dirichlet問題の解の一意性から $u(x)$ の値は $-\Delta u$ と $u|_{\pOm}$ のみから一意的に定められるが、Greenの表現公式は $u(x)$ の値を表現するためにこれらに加えて $Du\cdot\nu$ の値を要する。

この問題を解消するために、Greenの表現公式を次のように変形する。

$h\in C^2(\Ombar)$ を調和関数とすると部分積分により $u\in C^2(\Ombar)$ について $$\int_\Omega h(-\Delta u)=-\int_\pOm(hDu\cdot\nu-(Dh\cdot\nu)u)d\H^{n-1}.$$ $G(x,y)\colon=\Gamma(x-y)-h(y)$ とするとGreenの表現公式から $$u(x)=\int_\Omega G(x,y)(-\Delta u(y))dy+\int_\pOm (G(x,y)Du(y)\cdot\nu(y)-(D_yG(x,y)\cdot\nu(y))u(y))d\H^{n-1}(y).$$ ここで $h=h_x\in C^2(\Ombar)$ を $-\Delta h=0$ in $\Omega$ かつ $h(y)=\Gamma(x-y)$ for $y\in\pOm$ となるように選ぶことができれば $$u(x)=\int_\Omega G(x,y)(-\Delta u(y))dy+\int_\pOm (D_yG(x,y)\cdot\nu(y))u(y)d\H^{n-1}(y)$$ となる。

定義 21 (Green関数)

$G=G(x,y)$ を $\Omega$ における(Dirichlet-)Green関数、あるいは第1種Green関数という。

系 12よりGreen関数は(存在すれば)一意的であり、存在すればPoisson方程式の非斉次Dirichlet問題 $$\left\{ \begin{align*} -\Delta u&=f& &\inn\ \Omega\\ u&=\varphi& &\on\ \pOm \end{align*} \right.$$ の解 $u\in C^2(\Ombar)$ の表示を得られたことになる。

実際には一般の $C^{0,1}$ 領域 $\Omega$ におけるGreen関数を求める方法はなく、Green関数を用いた解析は困難である。一方、ごく限られた単純な領域の場合はGreen関数を求めることができることがある。

半空間の場合

半空間 $\R^n_+$ におけるGreen関数は容易に与えることができる。実際、 $$h_x(y)=\Gamma(x^*-y)=\Gamma(x-y^*)$$ とすれば $-\Delta h_x=0$ in $\R^n_+$, $h_x(y)=\Gamma(x-y)$ for $y\in\partial\R^N_+$ となり、 $$G(x,y)=\Gamma(x-y)-\Gamma(x^*-y)=\Gamma(x-y)-\Gamma(x-y^*)$$ が半空間 $\R^n_+$ におけるGreen関数になっている。ここで $x^*\colon=(x',-x_n)$ は平面 $\partial\R^n_+$ について $x=(x',x_n)\in \R^{n-1}\times\R$ と対称な点である。

命題 22

$u\in C^2(\R^n_+)\cap C(\overline{\R^n_+})$ は $u=0$ on $\partial\R^n_+$、$-\Delta u\in L^\infty(\R^n_+)$ をみたし、$\supp u$ は有界であるとする。このとき $$u(x)=\int_{\R^n_+}G(x,y)(-\Delta u(y))dy\ \for\ x\in\R^n_+.$$ ここで $G(x,y)\colon=\Gamma(x-y)-\Gamma(x^*-y)$、$x^*\colon=(x',-x_n)$。

Proof.

$u\in C^2(\overline{\R^n_+})$ の場合は一般論から従う。

$u\in C^2(\R^n_+)\cap C(\overline{\R^n_+})$ の場合を示す。$R\gt 0$ を $\supp u\subset\overline{D},D=(-R,R)^{n-1}\times(0,R)$ となるようにとる。 $$|u(x)|\le \frac{1}{2}R|\Delta u|_0x_n\for x\in D$$ を示す。$v\in C^2(\overline{D})$ を $$v(x)\colon=\frac{1}{2}|\Delta u|_0x_n(R-x_n)$$ により定める。このとき $v\ge 0=u$ on $\partial D$、$\Delta v=-|\Delta u|_0\le\Delta u$ in $D$。比較原理より $$u\le v\ \inn D.$$ 同様に $u\ge -v$ in $D$ も成り立ち、 $$|u|\le v\le\frac{1}{2}R|\Delta u|_0x_n.$$ $\xi\in C^\infty([0,\infty))$ を $0\le\xi\le 1$ on $[0,\infty)$、$\xi(t)=0$ for $t\in[0,1]$、$\xi(t)=1$ for $t\in[2,\infty)$ となるようにとり、$\varepsilon\gt 0$ について $u_\varepsilon(x)\colon=\xi(\varepsilon^{-1}x_n)u(x)$ とする。$u_\varepsilon$ は $u_\varepsilon\in C^2_c(\R^n_+)$、$u_\varepsilon(x)=u(x)$ if $x_n\gt 2\varepsilon$ をみたす。

$y=(y',y_n)\in\R^{n-1}\times(0,\infty)$ について $\Delta u(y)=\xi(\varepsilon^{-1}y_n)\Delta u(y)+2\varepsilon^{-1}\xi'(\varepsilon^{-1}y_n)D_nu(y)+\varepsilon^{-2}\xi^{\prime\prime}(\varepsilon^{-1}y_n)u(y)$ であるから $x\in\R^n_+$ と $\varepsilon\in(0,\frac{x_n}{2})$ について \begin{align*} u(x)=u_\varepsilon(x)&=\int_{\R^n_+}G(x,y)(\xi(\varepsilon^{-1}y_n)\Delta u(y)+2\varepsilon^{-1}\xi'(\varepsilon^{-1}y_n)D_nu(y)+\varepsilon^{-2}\xi^{\prime\prime}(\varepsilon^{-1}y_n)u(y))dy\\ &=\int_{\R^n_+}\left(G(x,y)(\xi(\varepsilon^{-1}y_n)\Delta u(y)-\varepsilon^{-2}\xi^{\prime\prime}(\varepsilon^{-1}y_n)u(y))-2\varepsilon^{-1}D_{y_n}G(x,y)\xi'(\varepsilon^{-1}y_n)u(y)\right)dy. \end{align*} ここで $D_\varepsilon\colon=(-R,R)^{n-1}\times(0,2\varepsilon)$ とすると $y=(y',y_n)$ について $$|G(x,y)|=|\Gamma(x'-y',x_n-y_n)-\Gamma(x'-y',x_n+y_n)|\le y_n\int_{-1}^1|D_n\Gamma(x'-y',x_n-ty_n)|dt\le C|x_n-y_n|^{1-n}y_n,$$ $$|D_{y_n}G(x,y)|=|-D_n\Gamma(x'-y',x_n-y_n)-D_n\Gamma(x'-y',x_n+y_n)|\le C|x_n-y_n|^{1-n}$$ となることに注意すると $$\left|\int_{\R^n_+}G(x,y)(1-\xi(\varepsilon^{-1}y_n))\Delta u(y)dy\right|\le C\int_{D_\varepsilon}|x_n-y_n|^{1-n}y_n|\Delta u(y)|dy\to 0\ (\varepsilon\to +0),$$ \begin{align*} \left|\varepsilon^{-2}\int_{\R^n_+}G(x,y)\xi^{\prime\prime}(\varepsilon^{-1}y_n)u(y)\right|&\le|\xi^{\prime\prime}|_0\varepsilon^{-2}\int_{D_\varepsilon}|G(x,y)u(y)|dy\\ &\le C\varepsilon^{-2}\int_{D_\varepsilon}|x_n-y_n|^{1-n}y_n\cdot\frac{1}{2}R|\Delta u|_0y_ndy\\ &\le CR|\Delta u|_0\int_{D_\varepsilon}|x_n-y_n|^{1-n}dy\to 0\ (\varepsilon\to +0), \end{align*} \begin{align*} \left|\varepsilon^{-1}\int_{\R^n}D_{y_n}G(x,y)\xi'(\varepsilon^{-1}y_n)u(y))dy\right|&\le C|\xi'|_0\varepsilon^{-1}\int_{D_\varepsilon}|x_n-y_n|^{1-n}\cdot\frac{1}{2}R|\Delta u|_0y_ndy\\ &\le CR|\Delta u|_0\int_{D_\varepsilon}|x_n-y_n|^{1-n}dy\to 0\ (\varepsilon\to +0). \end{align*} これより $$u(x)=\int_{\R^n_+}G(x,y)\Delta u(y)dy$$ が従う。

注意

補題 19より $u\in C^1(\overline{\R^n_+})$ となる。

また $x\in\R^n_+$、$y\in\partial\R^n_+$ について $y_n=0$、$|x-y|=|x^*-y|$ に注意すると $$D_yG(x,y)\cdot\nu(y)=-D_{y_n}G(x,y)=D_n\Gamma(x-y)+D_n\Gamma(x^*-y)=\frac{2}{n\omega_n}|x-y|^{-n}x_n.$$

定義 23 (上半空間におけるPoisson核)

$$K(x,y)\colon=\frac{2}{n\omega_n}|x-y|^{-n}x_n$$ により定まる $\R^n_+\times\partial\R^n_+$ 上の関数 $K$ を上半空間 $\R^n_+$ におけるPoisson核という。$K$ は $\Delta_x K(x,y)=0$ をみたす。

定理 24 (Poisson積分公式)

$\varphi\in C(\partial\R^n_+)\cap L^\infty(\partial\R^n_+)$ とする。 $$u(x)\colon=\int_{\partial\R^n_+}K(x,y)\varphi(y)d\H^{n-1}(y)\ (x\in\R^n_+)$$ とおくと $u$ は次をみたす:

  • (1) $$u\in C^\infty(\R^n_+)\cap L^\infty(\R^n_+),\Delta u=0\ \inn\ \R^n_+.$$
  • (2) $\xi\in\partial\R^n_+$ について

$$\lim_{\substack{x\in\R^n_+,\\ x\to\xi}}u(x)=\varphi(\xi).$$

Proof.

$$\int_{\R^n_+}K(x,y)d\H^{n-1}(y)=1$$ を示す。$x=(0,x_n)$ としてよい。 \begin{align*} \int_{\R^n_+}K(x,y)d\H^{n-1}(y)&=\frac{2}{n\omega_n}\int_{\R^{n-1}}x_n(x_n^2+|z|^2)^{-\frac{n}{2}}dz\\ &=\frac{2}{n\omega_n}\int_{\R^{n-1}}(1^2+|w|^2)^{-\frac{n}{2}}dw\\ &=\frac{2(n-1)\omega_{n-1}}{n\omega_n}\int_0^\infty r^{n-2}(1+r^2)^{-\frac{n}{2}}dr. \end{align*} ここで $$\frac{(n-1)\omega_{n-1}}{n\omega_n}=\frac{\Gamma\left(\frac{n}{2}\right)}{\Gamma\left(\frac{n-1}{2}\right)\sqrt{\pi}}=\frac{1}{B\left(\frac{n-1}{2},\frac{1}{2}\right)}.$$ ここで、$\Gamma$ は通常のgamma関数であり、$B$ はbeta関数である。 一方 $t=\frac{r^2}{1+r^2}$ と変数変換すると $$2\int_0^\infty r^{n-2}(1+r^2)^{-\frac{n}{2}}dr=\int_0^1 t^\frac{n-3}{2}(1-t)^\frac{3}{2}\cdot(1-t)^{-2}dt=B\left(\frac{n-1}{2},\frac{1}{2}\right).$$ これより $\int_{\R^n_+}K(x,y)d\H^{n-1}(y)=1$ が従う。とくに $$|u(x)|\le\int_{\R^n_+}K(x,y)|\varphi|_0d\H^{n-1}(y)\le|\varphi|_0$$ となり $u\in L^\infty(\R^n_+)$。

帰納法により任意の多重指数 $\alpha\gt 0$ について $D_x^\alpha K(x,y)$ は $|\alpha|$ 次以下の多項式 $P_\alpha$ と $|\alpha|-1$ 次以下の多項式 $Q_\alpha$ を用いて $$D_x^\alpha K(x,y)=|x-y|^{-n-2|\alpha|}x_nP(x-y)+|x-y|^{-n-2|\alpha|+2}Q(x-y)$$ と表される。$|x'-y'|+x_n\le \sqrt{2}|x-y|$ に注意すると $$|D_x^\alpha K(x,y)|\le C(|x-y|^{-n-|\alpha|}x_n+|x-y|^{-n-\alpha+1})\le C(x_n+|x'-y'|)^{-n-\alpha+1}.$$ これより $$u\in C^\infty(\R^n_+),D^\alpha u(x)=\int_{\R^n_+} D_x^\alpha K(x,y)\varphi(y)d\H^{n-1}(y).$$ また $\Delta_x K(x,y)=0$ より $$\Delta u=0\ \inn\ R^n_+.$$ (2)を示す。$\xi=(\xi',0)\in\partial\R^n_+$ とする。$\varepsilon\gt 0$ とし、$x=(x',x_n)\in\R^n_+$ は $|x'-\xi'|\lt\varepsilon$ をみたすとする。 $$\left|\int_{\partial\R^n_+\cap B_{2\varepsilon}(\xi)}K(x,y)(\varphi(y)-\varphi(\xi))d\H^{n-1}(y)\right|\le\sup_{\partial\R^n_+}|\varphi-\varphi(\xi)|\int_{\partial\R^n_+\cap B_{2\varepsilon}(\xi)}K(x,y)d\H^{n-1}(y)=\sup_{\partial\R^n_+\cap B_{2\varepsilon}(\xi)}|\varphi-\varphi(\xi)|.$$ 一方 $z\in\R^{n-1}$ について $|z-\xi|\gt 2\varepsilon\implies|z-x'|\lt\varepsilon$ となることに注意すると $$\left|\int_{\partial\R^n_+\backslash B_{2\varepsilon}(\xi)}K(x,y)(\varphi(y)-\varphi(\xi))d\H^{n-1}(y)\right|\le 2\sup_{\partial B}|\varphi|\int_{\{z\in\R^{n-1}\colon|z-x'|\gt\varepsilon\}}\frac{2}{n\omega_n}x_n|z-y'|^{-n}dz\le C\varepsilon^{-1}\sup_{\partial\R^n_+}|\varphi|x_n.$$ これより $$|u(x)-\varphi(\xi)|=\left|\int_{\partial\R^n_+}K(x,y)(\varphi(y)-\varphi(\xi))d\H^{n-1}(y)\right|\le\sup_{\partial\R^n_+\cap B_{2\varepsilon}(\xi)}|\varphi-\varphi(\xi)|+C\varepsilon^{-1}\sup_{\partial\R^n_+}|\varphi|x_n$$ となり $$\limsup_{\substack{x\in\R^n_+,\\ x\to\xi}}|u(x)-\varphi(\xi)|\le\sup_{\partial\R^n_+\cap B_{2\varepsilon}(\xi)}|\varphi-\varphi(\xi)|.$$ $\varepsilon\to +0$ として(2)を得る。

球の場合

$\Omega=B$ が球の場合も $B$ におけるGreen関数を具体的に与えることができる。

$B=B_R(0)$ とする。$x\in B\backslash\{0\}$ について $\overline{x}\colon=\frac{R^2}{|x|^2}x\in\R^n\backslash\overline{B}$ を球面 $\partial B$ について $x$ を反転した点とする。$a\gt 0$ を定数として $h_x(y)\colon=\Gamma(a|x-y|)$ とすると $\Delta h_x=0$ が成り立つ。また $|\overline{x}|^2=\frac{R^4}{|x|^2}$ に注意すると $y\in\partial B$ について $$|\overline{x}-y|^2=\frac{R^4}{|x|^2}-2\frac{R^2}{|x|^2}x\cdot y+R^2=\frac{R^2}{|x|^2}(|x|^2-2x\cdot y+R^2)=\frac{R^2}{|x|^2}|x-y|^2.$$ よって $a=\frac{|x|}{R}$ とすれば $h_x(y)=\Gamma(x-y)$ for $y\in\partial B$ が成り立つ。

一方 $h_0\colon=\Gamma(R)$ とすれば明らかに $\Delta h_0=0$ for $y\in B$、$h_0(y)=\Gamma(-y)$ for $\partial B$ が成り立つ。

従って $$G(x,y)\colon=\begin{cases} \Gamma(x-y)-\Gamma\left(\frac{|x|}{R}(\overline{x}-y)\right)&\jf x\in B\backslash\{0\}\\ \Gamma(y)-\Gamma(R)&\jf x=0 \end{cases}$$ によって $B$ におけるGreen関数が与えられる。

また $x\in B\backslash\{0\}$、$y\in\partial B$ について \begin{align*} D_yG(x,y)\cdot\nu(y)&=-\frac{1}{n\omega_n}\left(\left(\frac{|x|}{R}\right)^{2-n}|\overline{x}-y|^{-n}(\overline{x}-y)-|x-y|^{-n}(x-y)\right)\cdot\frac{y}{R}\\ &=\frac{1}{n\omega_n}|x-y|^{-n}\left(\frac{|x|^2}{R^2}(\overline{x}-y)-(x-y)\right)\cdot\frac{y}{R}\\ &=\frac{1}{n\omega_n}|x-y|^{-n}\left(1-\frac{|x|^2}{R^2}\right)y\cdot\frac{y}{R}\\ &=\frac{1}{n\omega_nR}|x-y|^{-n}(R^2-|x|^2). \end{align*} これは $x=0$ の場合も明らかに成り立つ。

定義 25 (球におけるPoisson核)

$$K(x,y)\colon=\frac{1}{n\omega_nR}|x-y|^{-n}(R^2-|x|^2)$$ により定まる $B_R(0)\times\partial B_R(0)$ 上の関数 $K$ を球 $B_R(0)$ におけるPoisson核という。

$K$ は $\Delta_x K(x,y)=0$ をみたし、$u\in C^2(\overline{B})$ を調和関数とするとPoisson積分公式 $$u(x)=\int_{\partial B}K(x,y)u(y)d\H^{n-1}(y)\ (x\in B)$$ が成り立つ。$u\in C^2(B)\cap C(\overline{B})$ の場合も $\varepsilon\gt 0$ について $u_\varepsilon(x)\colon=u((1-\varepsilon)x)$ として $u_\varepsilon$ に対して公式を適用して $\varepsilon\to +0$ とすることで同様の公式が成り立つ。

注意

Poisson積分公式により、Harnackの不等式をより精密にすることができる:非負値の調和関数 $u\in C^2(B)\cap C(\overline{B})$ と $x\in B$ について $$u(x)\le\frac{1}{n\omega_nR}\int_{B}(R-|x|)^{-n}(R^2-|x|^2)u(y)dy=\frac{R^{n-2}(R+|x|)}{(R-|x|)^{n-1}}u(0)$$ かつ $$u(x)\ge\frac{1}{n\omega_nR}\int_{B}(R+|x|)^{-n}(R^2-|x|^2)u(y)dy=\frac{R^{n-2}(R-|x|)}{(R+|x|)^{n-1}}u(0)$$ が成り立つ。とくに $r\in[0,R)$ について $$\sup_{B_r(0)}u\le\left(\frac{R+r}{R-r}\right)^n\inf_{B_r(0)}u.$$

系 26 (Liouvilleの定理)

$u\in C^2(\R^n)$ を上または下に有界な調和関数とすると $u$ は定数関数。

Proof.

$u\ge 0$ の場合を示せば十分。任意の $R\gt r\gt 0$ について精密化したHarnackの不等式より $\displaystyle \sup_{B_r(0)}u\le\left(\frac{R+r}{R-r}\right)^n\inf_{B_r(0)}u$。$R\to +\infty$ とすれば $\displaystyle \sup_{B_r(0)}u\le\inf_{B_r(0)}u$ を得る。これより $u$ は $B_r(0)$ 上で定数であり、$r\gt 0$ は任意であったから $u$ は定数関数である。

逆に与えられた $\varphi\in C(\partial B)$ についてPoisson積分公式によって $u=\varphi$ in $\partial B$ をみたす調和関数 $u\in C^2(B)\cap C(\overline{B})$ を構成できる。

定理 27 (Poisson積分公式)

$B=B_R(0)$ とし、$\varphi\in C(\partial B)$ とする。 $$u(x)\colon=\int_{\partial B}K(x,y)\varphi(y)d\H^{n-1}(y)\ (x\in B)$$ とおくと $u$ は次をみたす:

  • (1) $$u\in C^\infty(B),\Delta u=0\ \inn\ B.$$
  • (2) $\xi\in\partial B$ について

$$\lim_{\substack{x\in B,\\ x\to\xi}}u(x)=\varphi(\xi).$$

Proof.

$u\in C^\infty(B)$ は明らか。また $\Delta_x K(x,y)=0$ より $$\Delta u(x)=\int_{\partial B}\Delta_xK(x,y)\varphi(y)dy=0.$$ (2)を示す。$v\equiv 1$ in $B$ とすると明らかに $v\in C^2(\overline{B})$、$\Delta v=0$ in $B$ となるので $$\int_{\partial B}K(x,y)d\H^{n-1}(y)=1\ (x\in B).$$ $\xi\in\partial B$ とする。$\varepsilon\gt 0$ とし、$x\in B$ は $|x-\xi|\lt\varepsilon$ をみたすとする。 $$\left|\int_{\partial B\cap B_{2\varepsilon}(\xi)}K(x,y)(\varphi(y)-\varphi(\xi))d\H^{n-1}(y)\right|\le\sup_{\partial B\cap B_{2\varepsilon}(\xi)}|\varphi-\varphi_\xi|\int_{\partial B}K(x,y)dy=\sup_{\partial B\cap B_{2\varepsilon}(\xi)}|\varphi-\varphi_\xi|.$$ また $|y-\xi|\gt 2\varepsilon\implies|x-y|\gt\varepsilon$ となることに注意すると $$\left|\int_{\partial B\backslash B_{2\varepsilon}(\xi)}K(x,y)(\varphi(y)-\varphi(\xi))d\H^{n-1}(y)\right|\le 2\sup_{\partial B}|\varphi|\int_{\partial B}\varepsilon^{-n}(R^2-|x|^2)d\H^{n-1}(y)\le C\varepsilon^{-n}\sup_{\partial B}|\varphi|(R^2-|x|^2).$$ これより $$|u(x)-\varphi(\xi)|=\left|\int_{\partial B}K(x,y)(\varphi(y)-\varphi(\xi))d\H^{n-1}(y)\right|\le\sup_{\partial B\cap B_{2\varepsilon}(\xi)}|\varphi-\varphi_\xi|+C\varepsilon^{-n}\sup_{\partial B}|\varphi|(R^2-|x|^2)$$ となり、$|x|\to R\ (x\in B,x\to\xi)$ より $$\limsup_{\substack{x\in B,\\x\to\xi}}|u(x)-\varphi(\xi)|\le\sup_{\partial B\cap B_{2\varepsilon}(\xi)}|\varphi-\varphi_\xi|.$$ $\varepsilon\to +0$ として(2)を得る。

定理 28 (球におけるPoisson方程式のDirichlet問題の可解性)

$B\subset\R^n$ を開球とし、$f\in C^{0,\alpha}(B)\cap L^\infty(B)$、$\alpha\in(0,1)$、$\varphi\in C(\partial B)$ とするとDirichlet問題 $$\left\{\begin{align*} \Delta u&=f& &\inn\ B\\ u&=\varphi& &\on\ \partial B \end{align*}\right.$$ の解 $u\in C^2(B)\cap C(\partial B)$ が一意的に存在する。

Proof.

$B=B_R(0)$ としてよい。定理 20より $\Gamma*f\in C^2(B)\cap C^1(\overline{B})$、$\Delta(\Gamma*f)=f$ in $B$。また $K(x,y)$ を $B$ におけるPoisson核として $$v(x)\colon=\int_{\partial B}K(x,y)(\varphi(y)-\Gamma*f(y))d\H^{n-1}(y)$$ とすると定理 27より $v\in C^2(B)\cap C(\overline{B})$、$\Delta v=0$ in $B$、$v=\varphi-\Gamma*f$ on $\partial B$。従って $$u(x)=\Gamma*f(x)+\int_{\partial B}K(x,y)(\varphi(y)-\Gamma*f(y))d\H^{n-1}(y)$$ とすれば $u\in C^2(B)\cap C(\overline{B})$ は $\Delta u=f$ in $B$、$u=\varphi$ on $\partial B$ をみたす。

球におけるDirichlet問題の可解性から、さらなる調和関数の性質を得られる。

命題 29 (Schwarzの折り返し原理)

$\Omega\subset\R^n_+$ とし、$T\subset\pOm$ は相対開集合で $T\subset\partial\R^n_+$ をみたすとする。$u\in C^2(\Omega)\cap C(\Omega\cup T)$ は調和関数で $u=0$ on $T$ をみたすとする。$\Omega^*\colon=\{x^*=(x_1,\ldots,x_{n-1},-x_n)\colon x=(x_1,\ldots,x_n)\in\Omega\}$ として $D\colon=\Omega\cup T\cup\Omega^*$ とする。$v\in C(D)$ を $$v(x)\colon=\begin{cases} u(x)&\jf x\in\Omega\cup T\\ -u(x^*)&\jf x\in\Omega^* \end{cases}$$ と定めると $v$ は $D$ 上の調和関数である。

Proof.

$v$ が各 $\xi\in T$ の近傍で調和関数になっていることを示せばよい。

$B=B(\xi)\rcpt D$ とする。$w\in C^2(B)\cap C(\overline{B})$ を $$\left\{\begin{align*} \Delta w&=0& &\inn B\\ w&=v& &\on \partial B \end{align*}\right.$$ の解とする。$v(x^*)=-v(x)$ に注意すると $w^*(x)\colon=-w(x^*)$ とおくと $w^*$ も同じDirichlet問題の解になるので、系 12より $w^*=-w$。とくに $x\in T$ について $w(x)=-w(x^*)=-w(x)$ となり $w=0$ on $T$。また $v=u$ on $\Omega\cup T$ より $w=u$ on $\partial \partial B\cap\Ombar$。これより $w=u$ on $\partial(B\cap\Omega)$ となり系 12より $w=u$ in $B\cap\Omega$。また $x\in B\cap\Omega^*$ については $w(x)=-w(x^*)=-u(x^*)=v(x)$ となる。従って $w=v$ in $B$ であり、$v$ は $B$ 上の調和関数である。

命題 30 (調和関数の特異点の除去)

$x_0\in\Omega$ とする。$u\in C^2(\Omega\backslash\{x_0\})$ は調和関数で $$|u(x)|=\begin{cases} o(|x-x_0|^{2-n})&\jf n\ge 3\\ o(|\log|x-x_0||)&\jf n=2 \end{cases}$$ をみたすとする。このとき $u(x_0)$ の値を $u$ が $\Omega$ 上の調和関数となるように定めることができる。

Proof.

$B=B_R(x_0)\rcpt\Omega$ とする。$v\in C^2(B)\cap C(\overline{B})$ を $$\left\{\begin{align*} \Delta v&=0& &\inn B\\ v&=u& &\on\partial B \end{align*}\right.$$ の解とする。$v=u$ in $B\backslash\{x_0\}$ を示せばよい。

$\varepsilon\gt 0$ について $$w_\varepsilon(x)\colon=\begin{cases} u(x)-v(x)-\varepsilon(|x-x_0|^{2-n}-R^{2-n})&\jf n\ge 3\\ u(x)-v(x)-\varepsilon(\log R-\log|x-x_0|)&\jf n=2 \end{cases}$$ とおく。$w$ は $\Delta w=0$ in $B\backslash\{x_0\}$、$w=0$ on $\partial B$ をみたす。

$v$ は $B$ 上で有界であることに注意すると、$r\in(0,R)$ を十分小さくとると $w_\varepsilon\lt 0$ on $\partial B_r(x_0)$ となり、$w_\varepsilon=0$ on $\partial B$ とあわせて $w\le 0$ on $\partial(B\backslash \overline{B}_r(x_0))$ となる。最大値原理より $$w_\varepsilon\lt 0\ \inn B\backslash \overline{B}_r(x_0).$$ これより $w_\varepsilon\lt 0$ in $B\backslash\{x_0\}$。$\varepsilon\to +0$ とすれば $u-v\le 0$ in $B\backslash\{x_0\}$ を得る。

$u\ge v$ in $B\backslash\{x_0\}$ も同様に示すことができる。

調和関数の微分の評価

定理 31 (調和関数の微分の内部評価)

$\Omega\neq\R^n$ とする。$u\in C^2(\Omega)\cap L^\infty(\Omega)$ を調和関数とする。多重指数 $\alpha$ について $m\colon=|\alpha|$ とすると $$|D^\alpha u|_{0;\Omega}^{(m)}\le n^me^{m-1}m!|u|_{0;\Omega}.$$ ここで $|\cdot|_{0;\Omega}^{(m)}$ は内部ノルム(Hölder空間の基本事項の定義43)である。とくに任意の $\Omega'\rcpt\Omega$ について $$[u]_{m;\Omega'}\le C_{n,m}\dist(\Omega',\pOm)^{-m}|u|_{0;\Omega}.$$

Proof.

$\Omega=B=B_R(0)$、$u\in C^2(\overline{B})$ として $|D^\alpha u(0)|\le n^me^{m-1}m!R^{-m}|u|_{0;B}$ を示せば十分。

$m$ に関する帰納法により示す。$m=1$ の場合は $i\in\{1,\ldots,n\}$ について $\Delta(D_iu)=D_i(\Delta u)=0$ に注意すると $$|D_iu(0)|=\frac{1}{\omega_nR^n}\left|\int_B D_iu\right|=\frac{1}{\omega_nR^n}\left|\int_{\partial B} u(x)\frac{x_i}{R}d\H^{n-1}\right|\le nR^{-1}\sup_{\partial B}|u|$$ となることから従う。

ある $m\ge 1$ について $|\alpha|=m$ なる多重指数 $\alpha$ に対し主張が成り立ったとし、$|\alpha|=m$、$i\in\{1,\ldots,n\}$ とする。$t\in(0,1)$、$r=tR$ とすると仮定より $$|D_iD^\alpha u(0)|\le n^me^{m-1}m!r^{-m}|D_iu|_{0;B_r(0)}.$$ また $x\in B_r(0)$ として $m=1$ の場合を $B$ を $B_{R-r}(x)$ にとりかえて用いると $$|D_iu(x)|\le n(R-r)^{-1}|u|_{0;B_{R-r}(x)}$$ となるので $$|D_iu|_{0;B_r(0)}\le n(R-r)^{-1}|u|_{0;B}.$$ 従って $$|D_iD^\alpha u(0)|\le n^{m+1}e^{m-1}m!(1-t)^{-1}t^{-m}R^{-m}|u|_{0;B}.$$ ここで $t$ を $(1-t)t^m$ が最大となるように選ぶ。すなわち $t=\frac{m}{m+1}$ でこのとき $$(1-t)^{-1}t^{-m}=\frac{(m+1)^{m+1}}{m^m}=(m+1)(1+m^{-1})^m\lt (m+1)e.$$ これより $$|D_iD^\alpha u(0)|\le n^{m+1}e^m(m+1)!R^{-(m+1)}|u|_{0;B}$$ を得る。

系 32 (調和関数の内部 $C^{k,\alpha}$ 評価)

$\Omega\neq\R^n$ とする。$u\in C^2(\Omega)$ を調和関数とする。$k\in\Zp$、$\alpha\in[0,1]$、$\sigma\in\R$ とすると $$|u|_{k,\alpha;\Omega}^{(\sigma)}\le C_{n,k,\alpha}|u|_{0;\Omega}^{(\sigma)}.$$

Proof.

$d_x\colon=\dist(x,\pOm)$ とする。定理 31Hölder空間の基本事項の定理46、59より $$|u|_{k,\alpha;\Omega}^{(\sigma)}\le C\sup_{x\in\Omega}d_x^\sigma|u|_{k,\alpha;B_{\frac{1}{2}d_x}(x)}'\le C\sup_{x\in\Omega}d_x^\sigma|u|_{0;B_{d_x}(x)}'\le C|u|_{0;\Omega}^{(\sigma)}.$$

系 33 (調和関数の解析性)

$u\in C^2(\Omega)$ を調和関数とすると $u$ は各点で解析的である。

Proof.

$\Omega=B=B_R(0)$、$u\in C^2(\overline{B})$ として $u$ が $0$ において解析的であることを示せば十分。

定理 31と同様に $z\in\partial B_1(0)$ と $m\in\Zp$ と $s\in[0,R)$ について $$|D_z^m u(sz)|\le n^me^{m-1}m!(R-s)^{-m}|u|_{0;B}.$$ $z\in\partial B_1(0)$ と $t\in(0,R)$ について $$u(tz)-\sum_{|\alpha|\le m}\frac{1}{\alpha!}D^\alpha u(0)(tz)^\alpha=\frac{1}{(m-1)!}\int_0^t(t-s)^{m-1}D_z^mu(x+sz)ds$$ であるから $$\left|u(tz)-\sum_{|\alpha|\le m}\frac{1}{\alpha!}D^\alpha u(0)(tz)^\alpha\right|\le\frac{1}{m!}t^m\sup_{s\in[0,t]}|D_z^mu(x+sz)|\le n^me^{m-1}(R-t)^{-m}t^m|u|_{0;B}.$$ とくに $t\lt\frac{R}{ne+1}$ とすれば $ne(R-t)^{-1}t\lt 1$ となり $$\left|u(tz)-\sum_{|\alpha|\le m}\frac{1}{\alpha!}D^\alpha u(0)(tz)^\alpha\right|\to 0\ (m\to\infty)$$ となる。従って $u$ は $0$ において解析的である。

系 34

$\{u_m\}_{m=1}^\infty\subset C^2(\Omega)\cap L^\infty(\Omega)$ は調和関数の列で $\displaystyle \sup_m |u_m|_{0;\Omega}\lt\infty$ をみたすとする。このとき部分列 $\{u_{m_k}\}_{k=1}^\infty$ であってある調和関数 $u\in C^2(\Omega)$ に広義一様収束するものが存在する。

Proof.

任意の球 $B\rcpt\Omega$ について $\displaystyle \sup_m[u_m]_{1;B}\le C_B\sup_m|u_m|_{0;\Omega}$ であるからHölder空間の基本事項の定理62より $\{u_m\}_m$ は $B$ 上で一様収束する部分列をもつ。

$\{B_j\}_{j=1}^\infty$ を $B_j\rcpt\Omega$ なる球からなる $\Omega$ の開被覆とすると、対角線論法により $\{u_m\}_m$ は各 $B_j$ 上で一様収束する部分列 $\{u_{m_k}\}_k$ をもち、$\{u_{m_k}\}_k$ はある $u\in C(\Omega)$ に広義一様収束している。系 6より $u$ は調和関数である。

Dirichlet境界値問題の可解性

ここではDirichlet問題 $$\left\{\begin{align*} \Delta u&=0& &\inn\Omega\\ u&=\varphi& &\on\partial\Omega \end{align*}\right.\tag{$D_{\Delta}$}\label{DL}$$ の解をPerron法と呼ばれる方法で構成する。ここで $\varphi\in C(\pOm)$ である。Perron法とは方程式の解を「最大の劣解」として構成する方法であり、2階楕円型方程式の解の構成に広く使える初等的な解の構成である。(比較原理より(\ref{DL})の解 $u$ が存在すれば $v\le\varphi$ on $\pOm$ をみたす任意の劣調和関数 $v$ について $v\le u$ in $\Omega$ となることに注意せよ。)

注意

(\ref{DL})が任意の $\varphi\in C(\pOm)$ に対して解 $u$ をもてば、Poisson方程式のDirichlet問題 $$\left\{\begin{align*} \Delta u&=f& &\inn\Omega\\ u&=\varphi& &\on\partial\Omega \end{align*}\right.$$ も任意の $f\in C^{0,\alpha}(\Omega)\cap L^\infty(\Omega)$、$\alpha\in(0,1)$ と $\varphi\in C(\pOm)$ に対して解 $u$ をもつ。実際、定理 20より $v$ を $$\left\{\begin{align*} \Delta v&=0& &\inn\Omega\\ v&=\varphi-\Gamma*f& &\on\partial\Omega \end{align*}\right.$$ の解として $u=\Gamma*f+v$ とすればよい。

この節では $\Omega$ は有界かつ連結であるとする。準備として、劣(優)調和関数の定義を連続関数について定義できるように拡張する。

補題 35

$u\in C^2(\Omega)$ について次は同値:

  • (i) $u$ は劣(優)調和関数である。
  • (ii) 任意の開球 $B\rcpt\Omega$ と $h\ge(\le)u$ on $\partial B$ をみたす $B$ 上の調和関数 $h$ について $u\le(\ge) h$ in $B$。
Proof.

(i) $\implies$ (ii)は比較原理から直ちに従う。

$u$ が劣調和関数でなかったとする。$x_0\in\Omega$ であって $\Delta u(x_0)\lt 0$ となるものが存在する。

$B=B(x_0)\rcpt\Omega$ を十分小さくとると $\Delta u\lt 0$ in $B$ となる。$h$ を $h=u$ on $\partial B$ なる $B$ 上の調和関数とすると比較原理より $u\gt h$ in $B$ となる。これより $u$ は(ii)をみたさない。

定義 36 (劣(優)調和関数の定義の拡張)

$u\in C(\Omega)$ が任意の開球 $B\rcpt\Omega$ と $h\ge(\le)u$ on $\partial B$ をみたす $B$ 上の調和関数 $h$ について $u\le(\ge) h$ in $B$ をみたすとき $u$ は劣(優)調和関数であるという。

補題 37
  • (i) (強最大値原理、比較原理) $u\in C(\Omega)$ が劣調和関数、$v\in C(\Omega)$ が優調和関数であるとする。このとき $u-v$ は定数であるか、あるいは $\Omega$ において最大値をとらない。とくに $(u-v)^*\le 0$ on $\pOm$ とすると $u\le v$ in $\Omega$。
  • (ii) (調和持ち上げ) $\Omega'\subset\Omega$ を開集合とする。$u\in C(\Omega\backslash\Omega')$ が劣調和関数であるとき、$B\rcpt\Omega$ として $h\in C^2(\Omega')\cap C(\overline{\Omega'})$ を $h=u$ on $\pOm'\cap\Omega$ をみたす調和関数とすると

$$U\colon=\begin{cases} h& \on \Omega\backslash\Omega'\\ u& \inn\Omega' \end{cases}$$ により定まる $U\in C(\Omega)$ も劣調和関数である。$U$ を $u$ の $\Omega'$ 上での調和持ち上げという。

  • (iii) $u_1,\ldots,u_m\in C(\Omega)$ が劣調和関数であるとき $u\colon=\max\{u_1,\ldots,u_m\}$ も劣調和関数である。
Proof.

(i)を示す。$u-v$ がある $x_0\in\Omega$ で最大値 $M$ をとるとする。

任意の $B_R=B_R(x_0)\rcpt\Omega$ について $u-v=M$ in $B_R$ となることを示す。$r\in(0,R)$ とし、$h,k$ をそれぞれ $h=u,k=v$ on $\partial B_r$ をみたす調和関数とする。$u\le v+M$ より $h\le k+M$ on $\partial B_r$ で、$k(x_0)+M\le v(x_0)+M=u(x_0)\le h(x_0)$ となるので強最大値原理より $h=k+M$ in $B_r$。よって $u=v+M$ on $\partial B_r$ となり、$r\in(0,R)$ は任意であるから $u+v=M$ in $B_R$。

従って $\Omega_M\colon=\{x\in\Omega\colon u(x)-v(x)=M\}$ は開集合である。一方 $\Omega_M$ は明らかに $\Omega$ の相対位相について閉集合であり、また $x_0\in\Omega_M\neq\emptyset$。これより $\Omega_M=\Omega$ となり $u-v\equiv M$ in $\Omega$。

(ii)を示す。$B\rcpt\Omega$ を開球とし、$h,H$ をそれぞれ $h=u,H=U$ on $\partial B$ をみたす調和関数とする。$U\ge u$ in $\Omega$ より $H\ge h$ on $\partial B$ であるから $H\ge h\ge u$ in $B$。またこれより $H\ge U$ on $\partial \Omega'\cap B$ であり、$H=U$ on $\partial B\cap\Omega'$ とあわせて $H\ge U$ on $\partial(\Omega'\cap B)$ となるので $H\ge U$ in $\Omega'\cap B$。従って $U\le H$ in $B$ が成り立つ。

(iii)を示す。$B\rcpt\Omega$ を開球とし、$h_1,\ldots,h_m,h$ をそれぞれ $h_1=u_1,\ldots,h_m=u_m,h=u$ on $\partial B$ をみたす調和関数とする。$i=1,\ldots,m$ について、$u\ge u_i$ in $\Omega$ より $h\ge h_i$ on $\partial B$ であるから $h\ge h_i\ge u_i$ in $B$。これより $h\ge\max\{u_1,\ldots,u_m\}=u$ in $B$ を得る。

$\varphi\in C(\partial\Omega)$ とし、$S_\varphi$ を $u^*\le\varphi$ on $\pOm$ なる劣調和関数 $u\in C(\Omega)$ からなる $C(\Omega)$ の部分集合とする。

比較原理より直ちに次が成り立つ:

補題 38

$S_\varphi$ は次をみたす:

  • (i) 定数関数 $\displaystyle \inf_{\pOm}\varphi$ は $S_\varphi$ の元である。とくに $S_\varphi\neq\emptyset$。
  • (ii) 任意の $v\in S_\varphi$ について $\displaystyle v\le\sup_{\pOm}\varphi$ in $\Omega$。
定理 39

$$u\colon=\sup_{v\in S_\varphi}v$$ により $\Omega$ 上の関数 $u$ を定めると $$u\in C^2(\Omega),\Delta u=0\ \inn\Omega.$$

Proof.

$B=B(x_0)\rcpt\Omega$ を任意にとる。$\{u_m\}_{m=1}^\infty\subset S_\varphi$ を $u_m(x_0)\to u(x_0)\ (m\to\infty)$ となるようにとる。$u_m$ を $\displaystyle\max\left\{u_m,\inf_{\pOm}\varphi\right\}$ にとりかえて $\displaystyle u_m\ge\inf_\pOm\varphi$ in $\Omega$ としてよい。$U_m$ を $u_m$ の $B$ 上での調和持ち上げとする。$U_m\ge u_m$ in $\Omega$ で、$U_m$ も劣調和関数で $U_m=u_m\le\varphi$ on $\pOm$ をみたすので $U_m\in S_\varphi$ となり $U_m\le u$ in $\Omega$。とくに $u_m(x_0)\le U_m(x_0)\le u(x_0)$ であるから $U_m(x_0)\to u(x_0)\ (m\to\infty)$。

$\displaystyle \inf_\pOm\varphi\le u_m\le U_m\le\sup_{\pOm}\varphi$ であるから系 34より部分列に移って $U_m$ がある $B$ 上の調和関数 $w$ に $B$ 上で広義一様収束するとしてよい。$U_m\le u$ in $B$ より $w\le u$ in $B$。

$w=u$ in $B$ を示す。$w(y)\lt u(y)$ となる $y\in B$ が存在したとし、$\{\bar{u}_m\}_{m=1}^\infty\subset S_\varphi$ を $\bar{u}(y)\to u(y)\ (m\to\infty)$ となるようにとる。$V_m$ を $\max\{u_m,\bar{u}_m\}$ の $B$ 上での調和持ち上げとすると、$U_m$ と同様に $\bar{U}_m(x_0)\to u(x_0)$、$\bar{U}_m(y)\to u(y)\ (m\to\infty)$。再び部分列に移って $\bar{U}_m$ がある $B$ 上の調和関数 $\bar{w}$ に $B$ 上で広義一様収束するとしてよい。$U_m\le\bar{U}_m$ on $\partial B$ より $U_m\le\bar{U}_m$ in $B$ となるので $w\le\bar{w}$ in $B$。また $w(y)\lt u(y)=\bar{w}(y)$。強最大値原理より $w\lt\bar{w}$ in $B$ となるが、これは $w(x_0)=\bar{w}(x_0)=u(x_0)$ に矛盾。

従って $w=u$ in $B$ であり、$u$ は $B$ 上の調和関数である。$B$ は任意であったから、$u$ は $\Omega$ 上の調和関数となる。

従って、この $u$ が $u=\varphi$ on $\pOm$ をみたすかを調べればよい。

定義 40 (バリア関数,局所バリア関数)
  • (1) $\xi\in\pOm$ とする。

$$w_*\gt 0\ \inn\Ombar\backslash\{\xi\},\lim_{\substack{x\in\Omega,\\x\to\xi}}w(x)=0$$ をみたす優調和関数 $w\in C(\Omega)$ を $\xi$ におけるバリア関数という。

  • (2) $\xi\in\pOm$ とし、$U\subset\R^n$ を $\xi$ の開近傍とする。バリア関数の定義の $\Omega$ を $U\cap\Omega$ にとりかえたものをみたす $w\in C(U\cap\Omega)$ を $\xi$ における局所バリア関数という。
定義 41 (正則点)

$\xi\in\pOm$ におけるバリア関数が存在するとき $\xi$ は $\pOm$ の(ラプラシアンに関する)正則点であるという。

補題 42

$\xi\in\pOm$ を正則点とすると定理 39で構成した $u$ は $$\lim_{\substack{x\in\Omega,\\x\to\xi}}u=\varphi(\xi)$$ をみたす。

Proof.

$w$ を $\xi$ におけるバリア関数とする。$\varepsilon\gt 0$ とし、$r\gt 0$ を $|\varphi-\varphi(\xi)|\lt\varepsilon$ on $B_r(\xi)\cap\pOm$ となるようにとる。

$w_*$ は下半連続で $\pOm\backslash B_r(\xi)$ はコンパクトであるから $\displaystyle\inf_{\pOm\backslash B_r(\xi)}w_*\gt 0$。$v_+,v_-\in C(\Omega)$ を $$v_\pm\colon=\varphi(\xi)\pm\varepsilon\pm\left(\inf_{\pOm\backslash B_r(\xi)}w_*\right)^{-1}\sup_\pOm|\varphi-\varphi(\xi)|w$$ により定める。$v_+$ と $v_-$ はそれぞれ優調和関数、劣調和関数である。

$v_+$ と $v_-$ は $${v_-}^*\lt\varphi(\xi)-\varepsilon\lt\varphi\lt\varphi(\xi)+\varepsilon\lt {v_+}_*\ \on B_r(\xi)\cap\pOm$$ かつ $${v_-}^*\lt\varphi(\xi)-w_*^{-1}\cdot|\varphi-\varphi(\xi)|w_*\le\varphi(\xi)\le\varphi(\xi)+w_*^{-1}\cdot|\varphi-\varphi(\xi)|w_*\lt {v_+}_*\ \on\pOm\backslash B_r(\xi)$$ をみたす。

$v_-$ は ${v_-}^*\lt\varphi$ on $\pOm$ をみたす劣調和関数であるから $v_-\in S_\varphi$ であり $u\ge v_-$。また $v_+$ は ${v_+}_*\gt\varphi$ on $\pOm$ をみたす優調和関数であるから任意の $v\in S_\varphi$ について $v\le v_+$ であり $u\le v_+$。これより $$|u-\varphi(\xi)|\le\varepsilon+\left(\inf_{\pOm\backslash B_r(\xi)}w\right)^{-1}\sup_\pOm|\varphi-\varphi(\xi)|w\ \inn\Omega$$ を得る。とくに $$\limsup_{\substack{x\in\Omega,\\x\to\xi}}|u-\varphi(\xi)|\le\varepsilon.$$ $\varepsilon\to +0$ として主張を得る。

定理 43 (Laplace方程式のDirichlet問題の可解性の必要十分条件)

任意の $\varphi\in C(\pOm)$ についてDirichlet問題(\ref{DL})が解 $u\in C^2(\Omega)\cap C(\Ombar)$ をもつことは、$\pOm$ の任意の点が正則点であることと同値である。

Proof.

$\pOm$ の任意の点が正則点であるとすると、補題 42より定理 39の $u$ が(\ref{DL})の解になっている。

逆に任意の $\varphi\in C(\pOm)$ についてDirichlet問題(\ref{DL})が解 $u\in C^2(\Omega)\cap C(\Ombar)$ をもつとし、$\xi\in\pOm$ とすると、$w$ を $\varphi(x)=|x-\xi|$ に対する(\ref{DL})の解とすればこの $w$ は $\xi$ におけるバリア関数になっている。

従って、Laplace方程式のDirichlet問題はどのような点が正則点であるかという問題に帰着される。

補題 44

$\xi\in\pOm$ における局所バリア関数 $w\in C(U\cap\Omega)$ ( $U\subset\R^n$ は $\xi$ のある開近傍)が存在するとき $\xi$ は正則点である。

Proof.

$B=B(\xi)\rcpt U$ をとると $w_*$ は下半連続より $$m\colon=\inf_{(U\cap\Omega)\backslash B}w=\inf_{\overline{U\cap\Omega}\backslash B}w_*\gt 0.$$ このとき $\min\{w,m\}=m$ on $\partial U\cap\Omega$。

$\bar{w}\in C(\Omega)$ を $$\bar{w}\colon=\begin{cases} \min\{w,m\}& \on \overline{U}\cap\Omega\\ m&\inn \Omega\backslash\overline{U} \end{cases}$$ と定める。$w$ と定数関数 $m$ は優調和関数であるから $\bar{w}$ も優調和関数である。

また $\bar{w}_*\ge\min\{w^*,m\}$ on $\overline{U}\cap\Ombar$ かつ $\bar{w}_w=m$ on $\Ombar\backslash\overline{U}$ より $\bar{w}_*\gt 0$ on $\Ombar\backslash\{0\}$。一方、$\xi$ の十分小さい近傍において $w\lt m$ より $\bar{w}=w$ となるので $\displaystyle\lim_{\substack{x\in\Omega,\\x\to\xi}}\bar{w}(x)=0$ も成り立つ。

従って $\bar{w}$ は $\xi$ におけるバリア関数になっている。

$n=2$ の場合

$n=2$ の場合は複素対数関数を用いることで多くの場合にバリア関数を構成することができる。

定理 45

$n=2$ とする。$\xi\in\pOm$ とし、$\xi$ の開近傍 $U\subset\R^2$ と単連結開集合 $D\subset\R^2\backslash\{\xi\}$ で $U\cap\Omega\subset D$ となるものが存在するとき $\xi$ は正則点である。

Proof.

$\xi=0$ としてよい。

$\R^2$ を $\C$ と同一視する。Cauchy-Riemann方程式より $\R^2=\C$ の開集合上の複素正則関数の実部と虚部は調和関数となっていることに注意する。 定数 $a\gt 0$ を十分小さくとって $a|z|\lt 1$ in $D$ となるようにし、複素多価関数 $\log az$ の $D$ における枝 $f(z)$ をとると $f(z)\neq 0$ in $D$ となるので $$w(z)\colon=-\Re\frac{1}{f(z)}$$ は $D$ 上の調和関数である。また $w$ は $$w(z)=-\frac{\Re f(z)}{|f(z)|^2}=-\frac{\log a|z|}{|f(z)|^2}$$ と表され、$0\lt-\log a|z|\le|f(z)|$ より $$w\in C(\overline{D}),w\gt 0\on\overline{D}\backslash\{0\},\lim_{\substack{x\in D,\\x\to\xi}}w(0)=0$$ をみたす。$\overline{U\cap\Omega}\subset\overline{D}$ より $w$ は $0$ における局所バリア関数になっている。

$n\ge 3$ の場合

$n\ge 3$ の場合はより繊細である。例えば、次の外部錐条件をみたす $\xi\in\pOm$ は正則点である。

定義 46 (外部錐条件)

$\xi\in\pOm$ とする。錐 $C_\xi=\{x\in\R^n\colon |x-\xi|\le r,\nu\cdot(x-\xi)\ge c|x-\xi|\}\ (r\gt 0,\nu\in\partial B_1(0),c\in(0,1))$ であって $\Ombar\cap C_\xi=\{\xi\}$ をみたすものが存在するとき、$\xi$ は外部錐条件をみたすという。

相対開集合 $T\subset\pOm$ 上の各点が外部錐条件をみたすとき、$T$ は外部錐条件をみたすという。$\pOm$ が外部錐条件をみたすとき $\Omega$ は外部錐条件をみたすという。

$T$ が外部錐条件をみたし、かつ任意の $\xi\in T$ について $C_\xi$ を $C$ と合同にとれるような錐 $C$ が存在するとき $T$ は一様外部錐条件をみたすという。$\pOm$ が一様外部錐条件をみたすとき $\Omega$ は一様外部錐条件をみたすという。

定理 47

$\xi\in\pOm$ が外部錐条件をみたすとき $\xi$ は正則点である。

Proof.

$\xi=0$ としてよい。定義 46の $C$ をとって $U\colon=B_r(0)\cap\Omega$ とすると $x\in U$ について $\nu\cdot x\lt c|x|$。

$f\in C^2([-1,c])$、$\gamma\gt 0$ とし、 $$w(x)\colon=|x|^\gamma f\left(\frac{\nu\cdot x}{|x|}\right)\ (x\in U)$$ とする。 \begin{align*} D_i w(x)&=\gamma|x|^{\gamma-2}x_if+|x|^{\gamma-3}(\nu_i|x|^2-(\nu\cdot x)x_i)f',\\ D_{ij}w(x)&=(\gamma(\gamma-2)|x|^{\gamma-4}x_ix_j+\gamma\delta_{ij}|x|^{\gamma-2})f+((\gamma-1)|x|^{\gamma-3}(\nu_jx_i+\nu_ix_j)-(2\gamma-3)|x|^{\gamma-5}(\nu\cdot x)x_ix_j-\delta_{ij}|x|^{\gamma-3}(\nu\cdot x))f'+|x|^{\gamma-6}(\nu_i|x|^2-(\nu\cdot x)x_i)(\nu_j|x|^2-(\nu\cdot x)x_j)f^{\prime\prime} \end{align*} であり、$||x|^2\nu-(\nu\cdot x)x|^2=|x|^2(|x|^2-(\nu\cdot x)^2)$ より \begin{align*} \Delta w(x)&=\gamma(\gamma+n-2)|x|^{\gamma-2}f-(n-1)|x|^{\gamma-3}(\nu\cdot x)f'+|x|^{\gamma-4}(|x|^2-(\nu\cdot x)^2)f^{\prime\prime}\\ &=|x|^{\gamma-2}\left(\gamma(\gamma+n-2)f-(n-1)\left(\frac{\nu\cdot x}{|x|}\right)f'+\left(1-\left(\frac{\nu\cdot x}{|x|}\right)^2\right)f^{\prime\prime}\right). \end{align*} また $\theta_0\colon=\arccos c$ とし、$g(\theta)\colon=f(\cos\theta)\ (\theta\in[\theta_0,\pi])$ とすると \begin{align*} &\quad\gamma(\gamma+n-2)f(\cos\theta)-(n-1)\cos\theta f'(\cos\theta)+(1-\cos^2\theta)f^{\prime\prime}(\cos\theta)\\ &=\gamma(\gamma+n-2)f(\cos\theta)-(n-2)\cos\theta f'(\cos\theta)+(-\cos\theta f'(\cos\theta)+\sin^2\theta f^{\prime\prime}(\cos\theta))\\ &=\gamma(\gamma+n-2)g(\theta)+(n-2)\cot\theta g'(\theta)+g^{\prime\prime}(\theta). \end{align*} ここで、$\displaystyle\lim_{\theta\to\pi-0}\csc\theta\cot\theta=-\infty$ であるから $\displaystyle M\colon=(n-2)\sup_{\theta\in[\theta_0,\pi)}\csc\theta\cot\theta\lt\infty$。

$g\in C^\infty([\theta_0,\pi])$ を $$g(\theta)\colon=1+\int_{\theta_0}^\theta\left(\int_\sigma^\pi e^{M\cos\sigma-M\cos\tau}d\tau\right)d\sigma$$ により定める。このとき $$g'(\theta)=\int_\theta^\pi e^{M\cos\theta-M\cos\tau}d\tau,g^{\prime\prime}(\theta)=-M\sin\theta g'(\theta)-1,g^{\prime\prime\prime}(\theta)=(M\cos\theta+M^2\sin^2\theta)g'(\theta)-M\sin\theta.$$ $f(\cos\theta)=g(\theta)$ によって $f\in C^\infty((-1,c])\cap C([-1,c])$ を定める。

$g'(\pi)=0$ と $\csc\theta=(\pi-\theta)^{-1}+O(1)\ (\theta\to\pi-0)$ より $$f'(\cos\theta)=-\csc\theta g'(\theta)\to g^{\prime\prime}(\pi)\ (\theta\to\pi-0).$$ また $$f^{\prime\prime}(\cos\theta)=\csc^2\theta\cot\theta(g'(\theta)+\tan\theta g^{\prime\prime}(\theta)).$$ $g^{\prime\prime\prime}(\pi)=0$ であるから $\displaystyle g^{\prime\prime}(\theta)=g^{\prime\prime}(\pi)+\frac{1}{2}g^{\prime\prime\prime\prime}(\pi)(\pi-\theta)^2+O((\pi-\theta)^3)\ (\theta\to\pi-0)$。$\tan\theta=-(\pi-\theta)-\frac{1}{3}(\pi-\theta)^3+O((\pi-\theta)^5)\ (\theta\to\pi-0)$ とあわせて $$\tan\theta g^{\prime\prime}(\theta)=-g^{\prime\prime}(\pi)(\pi-\theta)-\left(\frac{1}{3}g^{\prime\prime}(\pi)+\frac{1}{2}g^{\prime\prime\prime\prime}(\pi)\right)(\pi-\theta)^3+O((\pi-\theta)^4)\ (\theta\to\pi-0).$$ これより $$g'(\theta)-\tan\theta g^{\prime\prime}(\theta)=\left(\frac{1}{2}g^{\prime\prime}(\pi)+\frac{1}{3}g^{\prime\prime\prime\prime}(\pi)\right)(\pi-\theta)^3+O((\pi-\theta)^4)\ (\theta\to\pi-0).$$ 従って $$f^{\prime\prime}(\cos\theta)\to\frac{1}{2}g^{\prime\prime}(\pi)+\frac{1}{3}g^{\prime\prime\prime\prime}(\pi)\ (\theta\to\pi-0)$$ となり $f\in C^2([-1,c])$。

また $(n-2)\cot\theta\le M\sin\theta$、$g'\ge 0$ より $$g^{\prime\prime}+(n-2)\cot\theta g'\le g^{\prime\prime}+M\sin\theta g'=-1.$$ とくに $\gamma$ を十分小さくとって $$\gamma(\gamma+n-2)\le|g|_{0;(\theta_0,\pi)}^{-1}$$ となるようにすればこの $g$ は $\gamma(\gamma+n-2)g(\theta)+(n-2)\cot\theta g'(\theta)+g^{\prime\prime}(\theta)\le 0$ をみたし、$\displaystyle w(x)=|x|^\gamma f\left(\frac{\nu\cdot x}{|x|}\right)$ により定まる $w$ は $\Delta w\le 0$ in $U$ をみたす。$w$ は $w\ge|x|^\gamma$、$w(0)=0$ をみたすのでこの $w$ は $0$ における局所バリア関数になっている。

従って $\Omega$ が外部錐条件をみたすとき任意の $\varphi\in C(\pOm)$ について(\ref{DL})は解 $u\in C^2(\Omega)\cap C(\Ombar)$ をもつ。

最後に、定理 47の証明で構成した局所バリア関数を用いると調和関数の大域Hölder評価の結果が得られる。

定理 48 (大域Hölder評価)

$\Omega$ が一様外部錐条件をみたすとする。次をみたす $\gamma=\gamma_\Omega\in(0,1]$ が存在する:

$\varphi\in C^{0,\alpha}(\pOm)$、$\alpha\in(0,1]$ について(\ref{DL})の解 $u\in C^2(\Omega)\cap C(\Ombar)$ は $$u\in C^{0,\frac{\alpha\gamma}{\alpha+\gamma}}(\Ombar),|u|_{0,\frac{\alpha\gamma}{\alpha+\gamma};\Omega}\le C_{\alpha,\Omega}|\varphi|_{0,\alpha;\pOm}$$ をみたす。

Proof.

$\varphi\not\equiv 0$ としてよい。

定理 47の証明より、$r_0\gt 0$、$M\gt m\gt 0$、$\gamma\in(0,1]$ が存在し各 $\xi\in\pOm$ におけるバリア関数 $w=w_\xi\in C(\overline{B_{r_0}(\xi)\cap\Omega})$ で $m|x-\xi|\le w(x)\le M|x-\xi|$ for $x\in B_{r_0}(\xi)\cap\Omega$ をみたすものが存在する。

$r\in(0,r_0]$ とする。補題 44の証明と同様に $$\bar{w}\colon=\begin{cases} \min\{w,mr^\gamma\}& \on \Bb_r(\xi)\cap\Omega\\ mr^\gamma&\inn \Omega\backslash\Bb_r(\xi) \end{cases}$$ によって $\xi$ におけるバリア関数 $w\in C(\Omega)$ が定まる。

$|\varphi-\varphi(\xi)|\le[\varphi]_{0,\alpha}r^\alpha$ in $\Bb_r(\xi)\cap\Omega$ に注意すると補題 42の証明より $$|u(x)-\varphi(\xi)|\le [\varphi]_{0,\alpha}r^\alpha+(mr^\gamma)^{-1}\sup_{\pOm}|\varphi-\varphi(\xi)|w(x)\le [\varphi]_{0,\alpha}r^\alpha+(\diam\Omega)^\alpha[\varphi]_{0,\alpha}\frac{M}{m}\left(\frac{|x-\xi|}{r}\right)^\gamma\ \for x\in \Bb_r(\xi)\cap\Omega.$$ ここで $\displaystyle r_1\colon=\min\left\{(\diam\Omega)^{-\frac{\alpha}{\gamma}}r_0^{1+\frac{\alpha}{\gamma}},r_0\right\}$ とする。$r\in[0,r_1]$ について $(\diam\Omega)^\frac{\alpha}{\alpha+\gamma}r^\frac{\gamma}{\alpha+\gamma}\le r_0$ が成り立つ。$x\in B_{r_1}(\xi)\cap\Omega$ として $r=(\diam\Omega)^\frac{\alpha}{\alpha+\gamma}|x-\xi|^\frac{\gamma}{\alpha+\gamma}$ ととると $r\in(0,r_0]$、$x\in \Bb_r(\xi)\cap\Omega$ となり $$|u(x)-\varphi(\xi)|\le[\varphi]_{0,\alpha}r^\alpha+(\diam\Omega)^\alpha[\varphi]_{0,\alpha}\frac{M}{m}\left(\frac{|x-\xi|}{r}\right)^\gamma\le C[\varphi]_{0,\alpha}|x-\xi|^\frac{\alpha\gamma}{\alpha+\gamma}.$$ $x,y\in\Omega$ は $\displaystyle d_x\le d_y\lt \frac{r_1}{2}$ をみたすとする。$\xi\in\pOm$ を $|x-\xi|=d_x$ をみたすようにとる。定理 31と $|z-\xi|\lt d_x$ for $z\in B_{\frac{d_x}{2}}(\xi)$ に注意すると $$|u(x)-u(y)|\le\begin{cases} [u-\varphi(\xi)]_{0,\frac{\alpha\gamma}{\alpha+\gamma};B_{\frac{d_x}{2}}(x)\cap\Omega}^{(0)}\left(\frac{d_x}{2}\right)^{-\frac{\alpha\gamma}{\alpha+\gamma}}|x-y|^\frac{\alpha\gamma}{\alpha+\gamma}\le C|u-\varphi(\xi)|_{0,\frac{\alpha\gamma}{\alpha+\gamma};B_{\frac{d_x}{2}}(x)\cap\Omega}d_x^{-\frac{\alpha\gamma}{\alpha+\gamma}}|x-y|^{-\frac{\alpha\gamma}{\alpha+\gamma}}\le C[\varphi]_{0,\alpha}|x-y|^\frac{\alpha\gamma}{\alpha+\gamma}&\jf |x-y|\lt\frac{d_x}{2}\\ |u(x)-\varphi(\xi)|+|u(y)-\varphi(\xi)|\le C[\varphi]_{0,\alpha}\left(d_x^\frac{\alpha\gamma}{\alpha+\gamma}+(d_x+|x-y|)^\frac{\alpha\gamma}{\alpha+\gamma}\right)\le C[\varphi]_{0,\alpha}|x-y|^\frac{\alpha\gamma}{\alpha+\gamma}&\jf |x-y|\ge\frac{d_x}{2}. \end{cases}$$ 従って $\Omega_1\colon=\left\{x\in\Omega\colon d_x\lt\frac{r_1}{2}\right\}$ とすると $[u]_{0,\frac{\alpha\gamma}{\alpha+\gamma};\Omega_1}\le C[\varphi]_{0,\alpha}$。

一方系 32より $\Omega_2\colon=\left\{x\in\Omega\colon d_x\gt\frac{r_1}{4}\right\}$ とすると $[u]_{0,\frac{\alpha\gamma}{\alpha+\gamma};\Omega_2}\le C|u|_0$。

Hölder空間の基本事項の命題9と最大値原理より $$|u|_{0,\frac{\alpha\gamma}{\alpha+\gamma};\Omega}\le C(|u|_0+[\varphi]_{0,\alpha})\le C|\varphi|_{0,\alpha}.$$

発散型の2階線型楕円型方程式

ここでは、発散型の楕円型微分作用素をもつ楕円型方程式を取り扱う。発散型の楕円型方程式を取り扱う際は、弱解と呼ばれる古典解より広い解の定義を用いることが多い。

弱解の定義

$L$ は発散型の楕円型微分作用素(\ref{DF})で、係数 $a_{ij}$、$b_i$、$c_i$、$d$ は $\Omega$ 上の可測関数であるとする。また $f_i,g\in L^1_\loc(\Omega)$、$\varphi_1\in W^{1,1}_\loc(\Omega)$ とする。また $T\subset\pOm$ を $C^{0,1}$ 境界部分( $\emptyset$ または $\pOm$ でもよい) とし、$\sigma$ を $T$ 上の $\H^{n-1}$ -可測関数、$\varphi_2\in L^1_\loc(T,\H^{n-1})$ とする。

次の方程式を考える: $$Lu=\sum_{i=1}^n D_i\left(\sum_{j=1}^n a_{ij}D_ju+b_iu\right)+\sum_{i=1}^n c_iD_iu+du=\sum_{i=1}^n D_if_i+g\ \inn\Omega.$$ 境界条件としては次の混合境界条件を考える: $$\left\{\begin{align*} u&=\varphi_1& &\on \Omega\backslash T\\ Nu&=\sum_{i=1}^n\nu_if_i+\varphi_2& &\on T \end{align*}\right..$$ ここで $N$ は $T$ 上の関数に作用する形式的な微分作用素 $$Nu\colon=\sum_{i,j=1}^n a_{ij}\nu_iD_ju+\sum_{i=1}^nb_i\nu_iu+\sigma u$$ である。

$T=\emptyset$ の場合は境界条件は $$u=\varphi_1\ \on\pOm$$ のみとし、これをDirichlet条件という。

$T=\pOm$ の場合は境界条件は $$Nu=\sum_i\nu_if_i+\varphi_2 \on\pOm$$ のみとし、これをRobin条件という。とくに $\sigma\equiv 0$ の場合をNeumann条件という。

命題 49
  • (1) $u\in L^1_\loc(\Omega)$ が任意の $v\in C^1_c(\Omega)$、$v\ge 0$ について $\int_\Omega uv\ge 0$ をみたすとき $u\ge 0$ a.e. in $\Omega$。
  • (2) $T\subset\pOm$ は相対開集合で任意の $T'\rcpt T$ が $\H^{n-1}(T)\lt\infty$ をみたすとする。$\varphi\in L^1_\loc(\Omega,\H^{n-1})$ が任意の $v\in C^1_c(\Omega\cup T)$、$v\ge 0$ について $\int_T \varphi vd\H^{n-1}\ge 0$ をみたすとき $\varphi\ge 0$ $\H^{n-1}$ -a.e. in $\Omega$。
Proof.

(1) を示す。$\eta_\varepsilon$ をmollifierとすると任意の $\Omega'\rcpt\Omega$ と十分小さい $\varepsilon\gt 0$ について $\eta_\varepsilon*u\ge 0$ in $\Omega'$ となるので $u\ge 0$ in $\Omega'$。これより主張が従う。

(2) を示す。$E\rcpt T$ を $\H^{n-1}$ -可測集合とする。$\H^{n-1}$ のBorel正則性(Hausdorff測度とarea_coarea_formulaの定理11)と仮定より $\H^{n-1}$ は $T$ 上のRadon測度である。Hausdorff測度とarea_coarea_formulaの命題8より $K_1\rcpt K_2\rcpt\ldots\rcpt E$ と相対開集合 $E \rcpt\ldots\rcpt T_2\rcpt T_1\rcpt T$ であって各 $j$ について $\H^{n-1}(T_j)\lt\H^{n-1}(E)+2^{-j}$、$\H^{n-1}(K_j)\gt\H^{n-1}(E)-2^{-j}$ をみたすものが存在する。開集合 $U_j\subset\R^n\backslash(\pOm\backslash T_j)$ を $U_j\cap\pOm=T_j$ となるようにとると、$\zeta_j\in C_c(\R^n\backslash(\pOm\backslash U_j))$ で $0\le\zeta_j\le 1$ in $\R^n\backslash(\pOm\backslash U_j)$ かつ $\zeta_j=1$ on $K$ をみたすものが存在する。この $\zeta$ は $\int_T \varphi\zeta_j d\H^{n-1}\ge 0$ をみたし、また $|\chi_E-\zeta_j|\le 1$ in $T_j$、$\chi_E-\zeta_j=0$ on $K_j\cup(T\backslash T_j)$ をみたすので $$\int_E \varphi d\H^{n-1}=\int_E \varphi d\H^{n-1}-\int_T\varphi\zeta_j d\H^{n-1}\ge -\int_T|\varphi||\chi_E-\zeta_j|d\H^{n-1}\ge -\int_{T_j\backslash K_j}|\varphi| d\H^{n-1}.$$ $\H^{n-1}(T_j-K_j)\to 0$ ( $j\to\infty$ )であるからLebesgueの収束定理より $\int_E\varphi d\H^{n-1}\ge 0$ を得る。これより主張が従う。

命題 50
  • (1) $L$ の係数と $u\in W^{1,1}_\loc(\Omega)$ は $\displaystyle \sum_{j}a_{i,j}D_ju+b_iu\in W^{1,1}_\loc(\Omega)$ ($ i=1,\ldots,n$ )、$\displaystyle\sum_i c_iD_iu+du\in L^1_\loc(\Omega)$ をみたすとする。また $f_i\in W^{1,1}_\loc(\Omega)$ とする。このとき次は同値:
    • (i) $\displaystyle Lu\ge(\le)\sum_i D_if_i+g$ in $\Omega$。
    • (ii)

$$\L(u,v)\colon=\int_\Omega\left(\sum_i\left(\sum_j a_{ij}D_ju+b_iu\right)D_iv-\left(\sum_i c_iD_iu+du\right)v\right)\le(\ge)\int_\Omega\left(\sum_i f_iD_iv-gv\right)\ (\forall v\in C^\infty(\Omega),v\ge 0) \tag{$\rm W_{sub}$}\label{wsub}.$$

  • (2) $L$ の係数と $u\in W^{2,1}_\loc(\OmT)$ は $\displaystyle \sum_{j}a_{i,j}D_ju+b_iu\in W^{1,1}_\loc(\OmT)$ ($ i=1,\ldots,n$ )、$\displaystyle\sum_i c_iD_iu+du\in L^1_\loc(\OmT)$ をみたすとする。また $f_i\in W^{1,1}_\loc(\OmT)$ とする。このとき次は同値:
    • (iii) $\displaystyle Lu\ge(\le)\sum_i D_if_i+g$ in $\Omega$、$\displaystyle Nu\le(\ge)\sum_i\nu_if_i+\varphi_2$ on $T$。
    • (iv)

$$\L(u,v)+\int_T \sigma uvd\H^{n-1}\le(\ge)\int_\Omega\left(\sum_i f_iD_iv-gv\right)+\int_T\varphi_2vd\H^{n-1}\ (\forall v\in C^1_c(\OmT),v\ge 0) \tag{$\rm W_{sub}'$}\label{wsub'}.$$

Proof.

()の外側の不等号についてのみ示す。

(1)を示す。(i)が成り立つとし、$v\in C^1_c(\Omega)$、$v\ge 0$ とすると部分積分により $$\L(u,v)=-\int_\Omega Luv\le-\int_\Omega\left(\sum_i D_if_i+g\right)v=\int_\Omega\left(\sum_i f_iD_iv-gv\right).$$ 逆に(ii)を仮定すると部分積分により任意の $v\in C^1_c(\Omega)$、$v\ge 0$ について $\displaystyle -\int_\Omega Luv\le -\int_\Omega\left(\sum_i D_if_i+g\right)v$ となり命題 49より(i)も成り立つ。

(2)を示す。(iii)が成り立つとし、$v\in C^1_c(\OmT)$、$v\ge 0$ とすると部分積分により \begin{align*} \L(u,v)+\int_T \sigma uvd\H^{n-1}&=-\int_\Omega Luv+\int_T \left(\sum_{i,j}a_{ij}\nu_iD_ju+\sum_i b_i\nu_iu+\sigma u\right)vd\H^{n-1}\\ &\le-\int_\Omega\left(\sum_i D_if_i+g\right)v+\int_T\left(\sum_i \nu_if_i+\varphi_2\right)vd\H^{n-1}\\ &=\int_\Omega\left(\sum_i f_iD_iv-gv\right)+\int_T\varphi_2vd\H^{n-1}. \end{align*} 逆に(iv)を仮定する。(\ref{wsub'})で $v\in C^1_c(\Omega)$ とすると(\ref{wsub})が得られることと(1)より $\displaystyle Lu\ge\sum_i D_if_i+g$ in $\Omega$。またこれより $v\in C^1_c(\OmT)$、$v\ge 0$ について \begin{align*} \int_T Nuvd\H^{n-1}&=\int_\Omega\left(\sum_i D_i\left(\sum_j a_{ij}D_juv+b_iuv\right)\right)+\int_T \left(\sigma uv-\sum_i\nu_if_i\right)\\ &=\int_\Omega\left(\sum_i\left(\sum_j a_{ij}D_ju+b_iu\right)D_iv-\left(\sum_i c_iD_iu+du\right)v\right)+\int_\Omega\left(\sum_iD_i\left(\sum_j a_{ij}D_ju+b_iu\right)+\left(\sum_i c_iD_iu+du\right)\right)v &=\L(u,v)+\int_\Omega Luv+\int_T \left(\sigma uv-\sum_i\nu_if_i\right)\\ &\le\L(u,v)+\int_\Omega\left(\sum_i D_if_i+g\right)v+\int_T \left(\sigma uv-\sum_i\nu_if_i\right)\\ &=\L(u,v)-\int_\Omega\left(\sum_i f_iD_iv-gv\right)+\int_T \sigma uv\\ &\le\int_T \varphi_2vd\H^{n-1}. \end{align*} 命題 49より(iii)を得る。

定義 51 (弱解)
  • (1) $f_i,g\in L^1_\loc(\Omega)$ とする。$u\in W^{1,1}_\loc(\Omega)$ が $\displaystyle \sum_j a_{ij}D_ju+b_iu\ (i=1,\ldots,n),\sum_i c_iD_iu+du\in L^1_\loc(\Omega)$ で(\ref{wsub})をみたすとき $u$ は(弱い意味で) $\displaystyle Lu\ge(\le)\sum_i D_if_i+g$ in $\Omega$ をみたすという。また $u$ が

$$\L(u,v)=\int_\Omega\left(\sum_i f_iD_iv-gv\right)\ (\forall v\in C^1_c(\Omega)) \tag{W}\label{wsol}$$ をみたすとき $u$ は(弱い意味で) $\displaystyle Lu=\sum_i D_if_i+g$ in $\Omega$ をみたす、あるいは方程式 $\displaystyle Lu=\sum_i D_if_i+g$ in $\Omega$ の弱解であるという。

  • (2) $u$ が $\displaystyle \sum_j a_{ij}D_ju+b_iu\ (i=1,\ldots,n),\sum_i c_iD_iu+du\in L^1_\loc(\OmT)$、$\sigma u\in L^1_\loc(T,\H^{n-1})$ かつ任意の $v\in C^1_c(\OmT)$、$v\ge 0$ について(\ref{wsub'})をみたすとき、$u$ は(弱い意味で) $\displaystyle Lu\ge(\le)\sum_iD_if_i-g$ in $\Omega$、$\displaystyle Nu\le(\ge)\sum_i\nu_if_i+\varphi_2$ on $T$ をみたすという。また $u$ が

$$\L(u,v)+\int_T \sigma uvd\H^{n-1}=\int_\Omega\left(\sum_i f_iD_iv-gv\right)+\int_T\varphi_2vd\H^{n-1}\ (\forall v\in C^1_c(\OmT))\tag{W'}\label{wsol'}$$ をみたすとき、$u$ は(弱い意味で) $\displaystyle Lu=\sum_iD_if_i-g$ in $\Omega$、$\displaystyle Nu=\sum_i\nu_if_i+\varphi_2$ on $T$ をみたすという。

  • (3) $\varphi_1\in W^{1,p}_\loc(\OmT)$ とする。$u\in W^{1,p}_\loc(\OmT)$ が $u-\varphi_1\in W^{1,p}_0(\OmT)$ をみたしかつ $\displaystyle Lu\ge(\le)\sum_iD_if_i-g$ in $\Omega$、$\displaystyle Nu=\sum_i\nu_if_i+\varphi_2$ on $T$ をみたすとき $u$ は( $W^{1,p}$ の意味で)境界値問題

$$\left\{\begin{align*} Lu&=\sum_iD_if_i+g & &\ \inn \Omega\\ u&=\varphi_1 & &\ \on \pOm\backslash T\\ Nu&=\varphi_2 & &\ \on T \end{align*}\right.\tag{MBP}\label{MBP}$$ の弱解であるという。

注意

(\ref{wsub})と(\ref{wsol})における $v$ は試験関数、あるいはテスト関数と呼ばれる。

$u\in W^{1,1}_\loc(\Omega)$ が $\displaystyle Lu\ge\sum_i D_if+g$ かつ $\displaystyle Lu\le\sum_i D_if+g$ をみたすとすると $u$ は $\displaystyle Lu=\sum_i D_if+g$ をみたす。実際、非負のテスト関数 $v$ については明らかに(\ref{wsol})が成り立ち、一般の場合も $v_+\in C^1_c(\Omega)$ を $v\ge 0$ in $\Omega$、$\displaystyle v_+=\sup_\Omega v$ on $\supp v$ となるようにとって $v_-=v_+-v$ とすると $v_+,v_-\ge 0$、$v=v_+-v_-$ となることから従う。

(3)で $p$ が明らかな場合は「 $W^{1,p}$ の意味で」をしばしば省略する。また $T=\emptyset$ の場合は単に $u\le(\ge,=)v$ on $\pOm$ という。

$\varphi_1\in W^{1,p}(\Omega)$ とすると $u\in W^{1,p}(\Omega)$ がDirichlet問題 $$\left\{\begin{align*} Lu&=\sum_i D_if_i+g& &\inn\Omega\\ u&=\varphi_1& &\on\pOm \end{align*}\right.\tag{DP}\label{DP}$$ の弱解となることは $u$ が $u-\varphi\in W^{1,p}_0(\Omega)$ かつ(\ref{wsol})をみたすことと同値である。

$\Omega$ が $C^{0,1}$ 領域であるとき $u\in W^{1,p}(\Omega)$ がRobin問題 $$\left\{\begin{align*} Lu&=\sum_i D_if_i+g& &\inn\Omega\\ Nu&=\sum_i\nu_if_i+\varphi_2& &\on\pOm \end{align*}\right.\tag{RP}\label{RP}$$ の弱解となることは $u$ が $$\L(u,v)+\int_\pOm \sigma uvd\H^{n-1}=\int_\Omega\left(\sum_i f_iD_iv-gv\right)+\int_\pOm \varphi_2vd\H^{n-1}\ (\forall v\in C^1(\Ombar))$$ をみたすことと同値である。

$$\overline{f}_i\colon=f_i-\sum_j a_{ij}D_j\varphi_1-b_i\varphi_1,\overline{g}\colon=g-\sum_i c_iD_i\varphi_1-d,\overline{\varphi}_2\colon=\varphi_2-\sum_{i,j} a_{ij}\nu_iD_j\varphi_1-\sum_i b_i\nu_i\varphi_1$$ とすると $u$ が(\ref{MBP})の弱解となることは $\overline{u}\colon=u-\varphi_1$ が $$\left\{\begin{align*} L\overline{u}&=\sum_iD_i\overline{f}_i+\overline{g} & &\ \inn \Omega\\ \overline{u}&=0 & &\ \on \pOm\backslash T\\ N\overline{u}&=\sum_iD_i\overline{f}_i+\overline{\varphi}_2 & &\ \on T \end{align*}\right.$$ の弱解となることと同値である。以下では多くの場合において $\varphi_1\equiv 0$ の場合のみを考える。

混合境界値問題の可解性

弱解には、関数解析の定理を援用して容易に存在を示すことができるという利点がある。とくに、次のLax-Milgramの定理とFredholmの択一性定理(Hilbert空間上の作用素論の定理13.13)を用いる。

定理 52 (Lax-Milgramの定理)

$H$ をHilbert空間とする。$\B\colon H\times H\to\R$ を有界強圧的双線型汎関数とする。すなわち以下をみたすとする:

  • (i) $\B$ は双線型である。
  • (ii)(有界性) 定数 $K\gt 0$ が存在し $|\B(u,v)|\le K\norm{u}_H\norm{v}_H\ (u,v\in H)$。
  • (iii)(強圧性) 定数 $\theta\gt 0$ が存在し $\B(u,u)\ge\theta\norm{u}_H^2\ (u\in H)$。

このとき $F\in H^*$ について $u\in H$ であって任意の $v\in H$ について $\B(u,v)=\pair{ F,v}_H$ をみたすものが一意的に存在する。

Proof.

$u\in H$ と $t\gt 0$ について(i)と(ii)より $(u,v)_H-t\B(u,v)+t\pair{ F,v}_H$ は $v\in H$ について線型かつ $$|(u,v)_H-t\B(u,v)+t\pair{ F,v}_H|\le ((tK+1)\norm{u}_H+t\norm{F}_{H^*})\norm{v}_H$$ となるので、Rieszの表現定理より $u\in H$ について $\Phi(u)\in H$ であって任意の $v\in H$ について $$(\Phi(u),v)_H=(u,v)_H-t\B(u,v)+t\pair{ F,v}_H$$ をみたすものが一意的に存在する。

この $\Phi\colon H\to H$ について、$u\in H$ が任意の $v\in H$ について $\B(u,v)=\pair{ F,v}_H$ をみたすことは $\Phi(u)=u$ と同値である。実際、$u$ が任意の $v\in H$ について $\B(u,v)=\pair{ F,v}_H$ をみたすとすると任意の $v\in H$ について $(\Phi(u),v)_H=(u,v)_H$ となるので $\Phi(u)=u$。逆に $\Phi(u)=u$ とすると任意の $v\in H$ について $(u,v)_H=(u,v)_H-t\B(u,v)+t\pair{ F,v}_H$ となるので $\B(u,v)=\pair{ F,v}_H$ が成り立つ。

$u,u',v\in H$ について $$(\Phi(u)-\Phi(u')-u+u',v)_H=-t\B(u-u',v)\le tK\norm{u-u'}_H\norm{v}_H$$ であるから $\norm{\Phi(u)-\Phi(u')-u+u'}_H\le tK\norm{u-u'}_H$ であり、 \begin{align*} \norm{\Phi(u)-\Phi(u')}_H^2&=\norm{\Phi(u)-\Phi(u')-u+u'}_H^2+2(\Phi(u)-\Phi(u'),u-u')_H-\norm{u-u'}_H^2\\ &=\norm{\Phi(u)-\Phi(u')-u+u'}_H^2+2\left(\norm{u-u'}_H^2-t\B(u-u',u-u')\right)-\norm{u-u'}_H^2\\ &\le(1-2t\theta+t^2K^2)\norm{u-u'}_H^2. \end{align*} $t$ を十分小さくとれば $1-2t\theta+t^2K^2\lt 1$ となり、Banachの不動点定理(位相空間論13:距離空間の位相(1)の定理13)より $\Phi(u)=u$ となる $u\in H$ が一意的に存在する。

注意

$G\colon H^*\to H$ を $GF=u$ により定めると、$G$ は全単射である。実際、$B\colon H\to H^*$ を $\pair{ Bu,v}_H\colon=\B(u,v)$ for $u,v\in H$ により定めると $B$ は $G$ の逆写像となっている。また $G$ は明らかに線型で、かつ $F\in H^*$ について $$\theta\norm{GF}_H^2\le\B(GF,GF)=\pair{F,GF}_H\le K\norm{F}_{H^*}\norm{GF}_H$$ をみたすので有界線型作用素である。

$\B$ が対称(すなわち任意の $u,v\in H$ について $\B(u,v)=\B(v,u)$ )のとき、$\B$ は $H$ に $(\cdot,\cdot)_H$ と同値なノルムを定める内積となり、$H$ は内積 $\B$ によってもHilbert空間となる。この場合のLax-Milgramの定理は内積 $\B$ に関するRieszの表現定理と同値であり、$u$ は汎関数 $$\frac{1}{2}\B(u,u)-\pair{ F,u}_H$$ が最小値をとる $u\in H$ として特徴づけられる。

以下では $\Omega$ と $T$ は次の条件をみたすとする: $$開集合D\subset\R^nとC^{0,1}境界部分S\subset\partial Dが存在し\Omega\subset D,T\rcpt S. \tag{$\diamondsuit$}\label{D}$$ また $\varphi_1\equiv 0$ であるとし、$a_{ij}$ は一様楕円性(\ref{UE})をみたし、$b_i,c_i,d,f_i,g,\sigma,\varphi_2$ は $$b,c\in L^q(\Omega)^n,d\in L^\frac{q}{2}(\Omega),\sigma\in L^{q-1}(T,\H^{n-1}),\begin{cases} q\in[n,\infty]&\jf n\ge 3\\ q\in(2,\infty]&\jf n=2 \end{cases}\ ,\tag{C1}\label{C1}$$ $$f\in L^2(\Omega)^n,g\in L^{2-\frac{4}{r+2}}(\Omega),\varphi_2\in L^{2-\frac{2}{r}}(T,\H^{n-1}),\begin{cases} r\in[n,\infty]&\jf n\ge 3\\ r\in(2,\infty]&\jf n=2 \end{cases}\tag{C2}\label{C2}$$ をみたすとする。ここで $b,c,f$ はベクトル値関数 $b=(b_1,\ldots,b_n),c=(c_1,\ldots,c_n),f=(f_1,\ldots,f_n)$ である。

さらに、$q\gt n$ のとき $$\mu\colon= \norm{\lambda^{-2}(|b|^2+|c|^2)+\lambda^{-1}|d|}_{\frac{q}{2};\Omega}+\norm{\lambda^{-1}\varphi}_{q-1;T}$$ とし、$q=n\ge 3$ のとき $$\mu\colon(0,\infty)\to[0,\infty),\mu(t)\colon=\norm{(\lambda^{-2}(|b|^2+|c|^2)+\lambda^{-1}|d|-t)^+}_{\frac{n}{2};\Omega}+\norm{(\lambda^{-1}|\sigma|-t)^+}_{n-1;T}$$ とおく。

命題 53
  • (1) $a_{ij},b_i,c_i,d,\sigma$ は(\ref{UE})、(\ref{C1})をみたすとする。このとき $u\in W^{1,2}_0(\OmT)$ と $v\in C^\infty_c(\OmT)$ について

$$\left|\L(u,v)+\int_T\sigma uvd\H^{n-1}\right|\le C_{q,\Lambda,\lambda,\mu,\Omega}\norm{u}_{1,2;\Omega}\norm{v}_{1,2;\Omega}.$$

  • (2) $f_i,g,\varphi_2$ は(\ref{C2})をみたすとする。このとき $v\in C^\infty_c(\OmT)$ について

$$\left|\int_\Omega\left(\sum_i f_iD_iv-gv\right)+\int_T \varphi_2vd\H^{n-1}\right|\le C_{r,\Omega}\left(\norm{f}_{2;\Omega}+\norm{g}_{2-\frac{4}{r+2};\Omega}+\norm{\varphi_2}_{2-\frac{2}{r};T}\right).$$

Proof.

Sobolevの不等式とトレースSobolev不等式(Sobolev空間とSobolevの不等式の定理67、命題84。トレースSobolev不等式については章末の注意も参照せよ。)より $u\in W^{1,2}(\OmT)$ と $v\in C^\infty_c(\OmT)$ について \begin{align*} \left|\L(u,v)\right|&\le\Lambda\norm{Du}_2\norm{Dv}_2+\norm{b}_q\norm{u}_{2+\frac{4}{q-2}}\norm{Dv}_2+\norm{c}_q\norm{Du}_2\norm{v}_{2+\frac{4}{q-2}}+\norm{d}_\frac{q}{2}\norm{u}_{2+\frac{4}{q-2}}\norm{v}_{2+\frac{4}{q-2}}\le C\norm{u}_{1,2}\norm{v}_{1,2},\\ \left|\int_T\sigma uvd\H^{n-1}\right|&\le\norm{\sigma}_{T;q-1}\norm{u}_{2+\frac{2}{q-2};T}\norm{v}_{2+\frac{2}{q-2};T}\le C\norm{u}_{1,2}\norm{v}_{1,2}. \end{align*} となり(1)を得る。また \begin{align*} \left|\int_\Omega f_iD_iu+\int_\Omega gu+\int_T \varphi_2ud\H^{n-1}\right|\le\norm{f}_2\norm{Du}_2+\norm{g}_{2-\frac{4}{r+2}}\norm{u}_{2+\frac{4}{r-2}}+\norm{\varphi_2}_{2-\frac{2}{r};T}\norm{u}_{1,2}\le C\norm{u}_{1,2} \end{align*} となり(2)を得る。

注意

(1)より $u,v\in W^{1,2}_0(\OmT)$ についても $\L(u,v)+\int_T \sigma uvd\H^{n-1}$ が定義でき、$W^{1,2}_0(\OmT)$ 上の有界双線型汎関数を定め、(1)が $u,v\in W^{1,2}_0(\OmT)$ についても成り立つ。また $\L(u,v)+\int_T \sigma uvd\H^{n-1}$ も $v\in W^{1,2}_0(\OmT)$ について連続で、$C^\infty_c(\OmT)$ は $W^{1,2}_0(\OmT)$ において稠密である。これより、$u\in W^{1,2}_0(\OmT)$ のときは(\ref{wsol'})はテスト関数 $v\in W^{1,2}_0(\OmT)$ についても成り立つ。

(2)より $\displaystyle \pair{F,v}_{(W^{1,2}_0(\OmT))^*}=\int_\Omega\left(\sum_i f_iD_iv-gv\right)+\int_T \varphi_2vd\H^{n-1}$ for $v\in W^{1,2}_0(\OmT)$ によって $F\in(W^{1,2}_0(\OmT))^*$ が定まる。

これらより混合境界値問題(\ref{MBP})は $F\in(W^{1,2}_0(\OmT))^*$ が与えられたとき $$\L(u,v)+\int_T \sigma uvd\H^{n-1}=\pair{F,v}_{W^{1,2}_0(\OmT)}\ (\forall v\in W^{1,2}_0(\OmT))\tag{$\rm \overline{W}$}\label{Wsol}$$ をみたす $u\in W^{1,2}_0(\OmT)$ を見つける問題に帰着される。

補題 54
  • (1) $q\gt n$ とする。$h\in L^\frac{q}{2}(\Omega)$、$\sigma\in L^{q-1}(\Omega)$ とすると $v\in W^{1,2}_0(\OmT)$、$\varepsilon\in(0,1)$ について

$$\left|\int_\Omega hv^2+\int_T \sigma v^2d\H^{n-1}\right|\le\varepsilon\int_\Omega |Dv|^2+C_{q,\Omega,T,\norm{h}_{\frac{q}{2};\Omega}+\norm{\sigma}_{q-1;T}}\varepsilon^{-\theta}\int_\Omega v^2,\theta=\theta_{n,q}\gt 0.\tag{E1}\label{E1}$$

  • (2) $n\ge 3$ とする。$h\in L^\frac{n}{2}(\Omega)$、$\sigma\in L^{n-1}(\Omega)$ とし、

$$\omega\colon(0,\infty)\to[0,\infty),\omega(t)\colon=\norm{(|h|-t)^+}_{\frac{n}{2};\Omega}+\norm{(\lambda^{-1}|\sigma|-t)^+}_{n-1;T}$$ とすると $$\left|\int_\Omega hv^2+\int_T\sigma v^2d\H^{n-1}\right|\le\varepsilon\int_\Omega |Dv|^2+C_{q,\Omega,T,\omega,\varepsilon}\int_\Omega v^2.\tag{E2}\label{E2}$$

Proof.

$$\left|\int_\Omega hv^2\right|\le C\norm{v}_{2+\frac{4}{q-2}}^2\le C\norm{v}_{1,2}^\frac{2n}{q}\norm{v}_2^{2-\frac{2n}{q}}\le\varepsilon\int_\Omega|Dv|^2+C\varepsilon^{-\frac{n}{q-n}}\int_\Omega v^2.$$ またSobolev空間とSobolevの不等式の定理79と補間不等式より \begin{align*} \left|\int_T \sigma v^2d\H^{n-1}\right|&\le C\norm{v}_{2+\frac{2}{q-2};T}^2\\ &\le C\left(\int_\Omega(|Dv|+|v|)|v|^{1+\frac{2}{q-2}}\right)^{1-\frac{1}{q-1}}\\ &\le C\norm{v}_{1,2}^{1-\frac{1}{q-1}}\norm{v}_{2+\frac{4}{q-2}}^{1+\frac{1}{q-1}}\\ &\le C\norm{v}_{1,2}^{1+\frac{n-1}{q-1}}\norm{v}_2^{1-\frac{n-1}{q-1}}\\ &\le \varepsilon\int_\Omega|Dv|^2+C\varepsilon^{-1-\frac{2n-2}{q-n}}\int_\Omega|v|^2. \end{align*} これらから(\ref{E1})が従う。

補題 55 (エネルギー評価)

$b_i,c_i,d,\sigma$ は(\ref{C1})をみたすとする。$u\in W^{1,2}_0(\OmT)$ について $$\L(u,u)+\int_T\sigma u^2d\H^{n-1}\ge \frac{\lambda}{4}\norm{u}_{1,2;\Omega}^2-C_{q,\Omega,T,\mu}\lambda\norm{u}_{2;\Omega}^2.$$

Proof.

補題 54とYoungの不等式より \begin{align*} \L(u,u)+\int_T \sigma u^2d\H^{n-1}&\ge\int_\Omega\left(\lambda|Du|^2-(|b|+|c|)|u||Du|-|d|u^2\right)-\int_T|\sigma|u^2d\H^{n-1}\\ &\ge\frac{\lambda}{2}\int_\Omega|Du|^2-\lambda\left(\int_\Omega\left((\lambda^{-2}(|b|^2+|c|^2)+\lambda^{-1}|d|)u^2\right)+\int_T\lambda^{-1}|\sigma|u^2d\H^{n-1}\right)\\ &\ge\frac{\lambda}{2}\int_\Omega|Du|^2-\lambda\left(\frac{1}{4}\int_\Omega|Du|^2+C\int_\Omega u^2\right)\\ &=\frac{\lambda}{4}\int_\Omega|Du|^2-C\lambda\int_\Omega u^2. \end{align*}

命題 56 (混合境界値問題の可解性1)

$b_i,c_i,d,\sigma$ は(\ref{C1})をみたすとする。$\gamma\in\R$ を十分大きくとると任意の $F\in(W^{1,2}_0(\OmT))^*$ について $u\in W^{1,2}_0(\OmT)$ であって $$\L(u,v)+\int_T \sigma uvd\H^{n-1}+\int_\Omega \gamma uv=\pair{F,v}_{W^{1,2}_0(\OmT)}\ (\forall v\in W^{1,2}_0(\OmT))\tag{$\rm \overline{W}_\gamma$}\label{Wsolg}$$ となるものが一意的に存在する。

Proof.

補題 55より十分大きな $\gamma$ について $$\L(u,u)+\int_T\sigma u^2d\H^{n-1}+\int_\Omega \gamma u^2\ge \frac{\lambda}{4}\norm{u}_{1,2;\Omega}^2$$ となり定理 52より主張が従う。

これより十分大きい $\gamma$ について混合境界値問題 $$\left\{\begin{align*} Lu-\gamma u&=\sum_iD_if_i+g & &\ \inn \Omega\\ u&=0 & &\ \on \pOm\backslash T\\ Nu&=\sum_i\nu_if_i+\varphi_2 & &\ \on T \end{align*}\right.$$ の弱解 $u\in W^{1,2}_0(\OmT)$ が一意的に存在する。

より詳細な可解性の結果をFredholmの択一性定理を用いて得ることができる。

定義 57 (包含 $L^2(\Omega)\subset(W^{1,2}_0(\OmT))^*$)

$u\in L^2(\Omega)$、$v\in W^{1,2}_0(\OmT)$ とすると明らかに $\displaystyle \int_\Omega uv\le\norm{u}_2\norm{v}_{1,2}$ であるから、$Iu\in (W^{1,2}_0(\OmT))^*$ を $\displaystyle \pair{ Iu,v}_{W^{1,2}_0(\OmT)}\colon=\int_\Omega uv$ for $v\in W^{1,2}_0(\OmT)$ によって定めることができ、$\norm{Iu}_{(W^{1,2}_0(\OmT))^*}\le\norm{u}_2$ となる。また $W^{1,2}_0(\OmT)\subset L^2(\Omega)$ が稠密であることから $u\in W^{1,2}_0(\OmT)$ が $Iu=0$ をみたすとすると $u=0$ となり、$I$ は単射である。そこで、$u$ を $Iu$ と同一視することにより連続な包含 $L^2(\Omega)\subset(W^{1,2}_0(\OmT))^*$ が定まる。

注意

Sobolev空間とSobolevの不等式の定義40も参照せよ。

内積構造を用いてRieszの表現定理により $(W^{1,2}_0(\OmT))^*=W^{1,2}_0(\OmT)$ と見做す同一視は多くの場合採用されない。Sobolev空間とSobolevの不等式の定義40の注意も参照せよ。

命題 58

定義 57で定義した包含 $L^2(\OmT)\subset(W^{1,2}_0(\OmT))^*$ について次が成り立つ:

  • (1) $W^{1,2}_0(\OmT)\subset(W^{1,2}_0(\OmT))^*$ は稠密である。
  • (2) 包含 $W^{1,2}_0(\OmT)\subset(W^{1,2}_0(\OmT))^*$ はコンパクトである。すなわち、$\{u_m\}_{m=1}^\infty\subset W^{1,2}_0(\OmT)$ が $\displaystyle \sup_m\norm{u_m}_{1,2;\Omega}\lt\infty$ をみたすとすると $\{u_m\}_{m=1}^\infty$ は $(W^{1,2}_0(\OmT))^*$ において収束する部分列をもつ。
Proof.

(1)を示す。$\Phi\in(W^{1,2}_0(\OmT))^{**}$ が任意の $v\in W^{1,2}_0(\OmT)$ について $\pair{\Phi,v}_{(W^{1,2}_0(\OmT))^*}=0$ をみたすとして $\Phi=0$ を示せばよい。$W^{1,2}_0(\OmT)$ は回帰的Banach空間であるから $u\in W^{1,2}_0(\OmT)$ であって任意の $v\in (W^{1,2}_0(\OmT))^*$ について $$\pair{ F,u}_{W^{1,2}_0(\OmT)}=\pair{\Phi,F}_{(W^{1,2}_0(\OmT))^*}$$ をみたすものが存在するが、仮定より任意の $v\in W^{1,2}_0(\OmT)$ について $\displaystyle\int_\Omega uv=0$ となり $u\equiv 0$。これより $\Phi=0$ となる。

(2)はRellich-Kondrachovの定理(Sobolev空間とSobolevの不等式の定理69)より包含 $W^{1,2}_0(\OmT)\subset L^2(\Omega)$ がコンパクトであることから従う。

定義 59 (形式的共役作用素、双対問題)

形式的な微分作用素 $L^*$ を $$L^*u=\sum_j D_j\left(\sum_i a_{ij}D_iu-c_ju\right)-\sum_i b_iD_iu+du$$ により定め、$L$ の形式的共役作用素という。また $T$ 上の関数に作用する形式的な微分作用素 $N^*$ を $$N^*u=\sum_{i,j} a_{ij}\nu_jD_iu-\sum_i c_i\nu_iu+\sigma_2u$$ と定め、混合境界値問題 $$\left\{\begin{align*} L^*v&=0& & \inn\Omega\\ v&=0& & \on\pOm\backslash T\\ N^*v&=0& &\on T \end{align*}\right.\tag{$\rm MBP_0^*$}\label{MBP*}$$ を斉次問題 $$\left\{\begin{align*} Lu&=0& & \inn\Omega\\ u&=0& & \on\pOm\backslash T\\ Nu&=0& &\on T \end{align*}\right.\tag{$\rm MBP_0$}\label{MBP0}$$ の双対問題という。

定義より $v\in W^{1,2}_0(\OmT)$ が(\ref{MBP*})の弱解であることは任意の $u\in W^{1,2}_0(\OmT)$ について $\displaystyle \L(u,v)+\int_T \sigma uvd\H^{n-1}=0$ をみたすことと同値である。

定理 60 (混合境界値問題の可解性2)

$|\Omega|\lt\infty$ とし、$b_i,c_i,d,\sigma$ は(\ref{C1})をみたすとする。次が成り立つ:

  • (1) 次は同値:
    • (i) 任意の $F\in(W^{1,2}_0(\OmT))^*$ について、(\ref{Wsol})をみたす $u\in W^{1,2}_0(\OmT)$ が存在する。
    • (ii) 混合境界値問題 (\ref{MBP0}) の弱解 $u\in W^{1,2}_0(\OmT)$ は $0$ のみである。
  • (2) (\ref{MBP0})の弱解のなす $W^{1,2}_0(\OmT)$ の部分空間を $M$、(\ref{MBP*})の弱解のなす $W^{1,2}_0(\OmT)$ の部分空間を $M^*$ とすると

$$\dim M=\dim M^*\lt\infty.$$

  • (3) $F\in(W^{1,2}_0(\OmT))^*$ について、次は同値:
    • (iii) (\ref{Wsol})をみたす $u\in W^{1,2}_0(\OmT)$ が存在する。
    • (iv) 任意の(\ref{MBP*})の弱解 $v\in W^{1,2}_0(\OmT)$ について

$$\pair{ F,v}_{W^{1,2}_0(\OmT)}=0.$$

Proof.

$\gamma\gt 0$ を命題 56の証明と同様にとると、$F\in (W^{1,2}_0(\OmT))^*$ に対して(\ref{Wsol})をみたす $u\in W^{1,2}_0(\OmT)$ を $GF$ とすることによって $G\colon (W^{1,2}_0(\OmT))^*\to W^{1,2}_0(\OmT)$ が定まる。定理 52の注意より $G$ は全単射有界線型作用素である。

包含 $W^{1,2}_0(\OmT)\subset(W^{1,2}_0(\OmT))^*$ はコンパクトであるから、$G$ は $(W^{1,2}_0(\OmT))^*$ 上のコンパクト作用素(Hilbert空間上の作用素論の定義13.1。定理13.10も参照せよ。)になっている。

また、$F\in (W^{1,2}_0(\OmT))^*$ と $u\in W^{1,2}_0(\OmT)$ について $$\text{(\ref{Wsol})}\iff u=G(F+\gamma u)\iff u-\gamma Gu=GF.$$ $R(G)=W^{1,2}_0(\OmT)$ であるから(i) $\iff W^{1,2}_0(\OmT)\subset R(I-\gamma G)$。ここで $I$ は恒等作用素である。 $W^{1,2}_0(\OmT)\subset(W^{1,2}_0(\OmT))^*$ は稠密であって、Hilbert空間上の作用素論の命題13.11より $R(I-\gamma G)\subset(W^{1,2}_0(\OmT))^*$ は閉であるから $${\rm (i)}\iff R(I-\gamma G)=(W^{1,2}_0(\OmT))^*.$$

Fredholmの択一性定理(Hilbert空間上の作用素論の定理13.13)より $${\rm (i)}\iff N(I-\gamma G)=\{0\}.$$

一方、(\ref{MBP})の弱解 $u$ は任意の $v\in W^{1,2}_0(\OmT)$ について $\displaystyle \L(u,v)+\int_T \sigma uvd\H^{n-1}=0$ をみたすので $u=\gamma Gu$。

逆に $F\in(W^{1,2}_0(\OmT))^*$ が $F-\gamma GF=0$ をみたすとすると $GF=\gamma^{-1}F$ となるので $F\in W^{1,2}_0(\OmT)$ で任意の $v\in W^{1,2}_0(\OmT)$ について $\displaystyle \L(\gamma^{-1}F,v)+\gamma^{-1}\int_T \sigma Fvd\H^{n-1}+\int_\Omega Fv=\int_\Omega Fv$ となる。これより $F$ は(\ref{MBP})の弱解である。これらより $M=N(I-\gamma G)$。

以上より(i) $\iff M=\{0\}$ であり、(1)が従う。

(2)を示す。$G^*\colon(W^{1,2}_0(\OmT))^*\to W^{1,2}_0(\OmT)$ を $G$ の共役作用素とする。$G^*$ の制限 $G^*\colon W^{1,2}_0(\OmT)\to W^{1,2}_0(\OmT)$ が $G\colon(W^{1,2}_0(\OmT))^*\to (W^{1,2}_0(\OmT))^*$ の共役作用素になっていることに注意する。

$u\in W^{1,2}_0(\OmT)$、$F=G^{-1}u\in (W^{1,2}_0(\OmT))^*$ とすると $H\in (W^{1,2}_0(\OmT))^*$ について $$\L(u,G^*H)+\int_T \sigma uG^*Hd\H^{n-1}+\int_\Omega \gamma uG^*H=\pair{ F,G^*H}_{W^{1,2}_0(\OmT)}=\pair{ H,u}_{W^{1,2}_0(\OmT)}.$$ とくに $v\in M^*$ は任意の $u\in W^{1,2}_0(\OmT)$ について $\displaystyle \int_\Omega\gamma uG^*v=\int_\Omega uv$ となり $\gamma G^*v=v$。これより $M^*\subset N(I-\gamma G^*)$。

逆に $H\in(W^{1,2}_0(\OmT))^*$ が $H-\gamma G^*H=0$ をみたすとすると $H\in W^{1,2}_0(\OmT)$、$G^*H=\gamma^{-1}H$ で、任意の $u\in W^{1,2}_0(\OmT)$ について $$\L(u,\gamma^{-1}H)+\int_T \gamma^{-1}\sigma uHd\H^{n-1}+\int_\Omega uH=\int_\Omega uH$$ となり $\displaystyle \L(u,H)+\int_T\sigma uHd\H^{n-1}=0$。これより $v\in M^*$ となり、$M^*=N(I-\gamma G^*)$。

Hilbert空間上の作用素論の命題13.14より $\dim M=\dim N(I-\gamma G)=\dim N(I-\gamma G^*)=\dim M^*\lt\infty$ が従う。

(3)を示す。(\ref{Wsol}) $\iff u-\gamma Gu=GF$ であったから、(iii)は $GF\in R(I-\gamma G)$ と同値である。一方、$R(I-\gamma G)\subset W^{1,2}_0(\OmT)$ は閉であるからHilbert空間上の作用素論の命題3.9の(5)より $R(I-\gamma G)=(N(I-\gamma G^*))^\bot={M^*}^\bot$。また $v\in M^*$ について $$\int_\Omega GFv=\pair{ F,G^*v}_{W^{1,2}_0(\OmT)}=\gamma^{-1}\pair{F,v}_{W^{1,2}_0(\OmT)}$$ となるので $GF\in {M^*}^\bot\iff F\in {M^*}^\bot$。以上より主張が従う。

弱解の大域有界性

ここでは、係数と非斉次項への適当な仮定のもと、(\ref{MBP})の弱解 $u$ は $\Omega$ 上で有界となることを示す。

$k\gt 0$ とし、 $$\bar{b}\colon=\lambda^{-2}(|b|^2+k^{-2}|f|^2),\bar{c}\colon=\lambda^{-2}|c|^2,\bar{d}\colon=\lambda^{-1}(d^-+k^{-1}g^-),\bar{\sigma}\colon=\lambda^{-1}(\sigma^-+k^{-1}\varphi_2^-)$$ とおく。

補題 61

$u\in W^{1,2}_0(\OmT)$ が $\displaystyle Lu\ge\sum_i D_if-g$ in $\Omega$、$\displaystyle Nu\le\sum_i\nu_if_i+\varphi_2$ on $T$ をみたすとする。$G(k)=0$、$G'(t)\gt 0$ if $t\gt k$ をみたす $G\in W^{1,\infty}(k,\infty)$ について、$H\in W^{1,\infty}(k,\infty)$ を $\displaystyle H(t)\colon=\int_0^t G'(s)^2ds$ と定めると $$\int_{\{u\gt k\}}|D(G(u))|^2\le C\left(\int_{\{u\gt k\}}\left(\bar{b}G'(u)^2u^2+\bar{c}\frac{H(w)^2}{G'(w)^2}+\bar{d}H(u)u\right)+\int_{T\cap\{u\gt k\}}\bar{\sigma}H(u)ud\H^{n-1}\right).\tag{E3}\label{E3}$$

Proof.

テスト関数として $H(w)$ を用いると \begin{align*} 0&\ge\int_{\{u\gt k\}}\left(\sum_i\left(\sum_j a_{ij}D_ju+b_iu-f\right)G'(u)^2D_iu-\left(\sum_ic_iu+du-g\right)H(u)\right)+\int_{T\cap\{u\gt k\}}\left(\sigma u-\varphi_2\right)H(u)d\H^{n-1}\\ &\ge\int_{\{u\gt k\}}\left(\lambda G'(u)^2|Du|^2-(|b|+k^{-1}|f|)G'(u)^2u|Du|-cH(u)|Du|-(d^-+k^{-1}g^-)H(u)u\right)-\int_{T\cap\{u\gt k\}}(\sigma^-+k^{-1}\varphi_2^-)H(u)d\H^{n-1}\\ &\ge\frac{\lambda}{2}\int_{\{u\gt k\}}|D(G(u))|^2-C\lambda\left(\int_{\{u\gt k\}}\left(\bar{b}G'(u)^2u^2+\bar{c}\frac{H(u)^2}{G'(u)^2}+\bar{d}H(w)u\right)+\int_{T\cap\{u\gt k\}}\bar{\sigma}H(u)ud\H^{n-1}\right) \end{align*} となり(\ref{E3})が成り立つ。

定理 62 (弱解の大域有界性1)

$b_i,c_i,d,\sigma,f_i,g,\varphi_2$ は $$b_i\in L^q(\Omega),c_i\in\begin{cases} L^n(\Omega)&\jf n\ge 3\\ L^q(\Omega)&\jf n=2 \end{cases},d\in L^\frac{q}{2}(\Omega),\sigma\in L^{q-1}(T,\H^{n-1}),q\gt n,\tag{C3}\label{C3}$$ $$f_i\in L^q(\Omega),g\in L^\frac{q}{2}(\Omega),\varphi_2\in L^{q-1}(T,\H^{n-1}),q\gt n,\tag{C4}\label{C4}$$ をみたすとし、 $$\mu_1\colon=\norm{\lambda^{-2}|b|^2+\lambda^{-1}d^-}_{\frac{q}{2};\Omega}+\norm{\lambda^{-1}\varphi_2}_{q-1;T}$$ とする。また $n\ge 3$ のとき $$\mu_2\colon(0,\infty)\to[0,\infty),\mu_2(t)\colon=\norm{(\lambda^{-2}|c|^2-t)^+}_{\frac{n}{2};\Omega}$$ とし、$n=2$ のときは $\mu_2\colon=\norm{\lambda^{-1}|c|}_{q;\Omega}$ とする。

$u\in W^{1,2}_0(\OmT)$ が $\displaystyle Lu\ge(\le,=)\sum_i D_if-g$ in $\Omega$、$Nu\le(\ge,=)\varphi_2$ on $T$ をみたすとすると $$\sup_{\Omega}u(-u,|u|)\le C_{q,\Omega,T,\mu_1,\mu_2}\left(\norm{u^+(u^-,u)}_{2;\Omega}+\lambda^{-1}\left(\norm{f}_{q;\Omega}+\norm{g}_{\frac{q}{2};\Omega}+\norm{\varphi_2}_{q-1;T}\right)\right).$$

Proof.

劣解の場合を示す。$k\gt 0$ を $$k\ge \lambda^{-1}\left(\norm{f}_{q;\Omega}+\norm{g}_{\frac{q}{2};\Omega}+\norm{\varphi_2}_{q-1;T}\right)$$ となるようにとる。 $$\norm{\bar{b}}_{\frac{q}{2};\Omega},\norm{\bar{d}}_{\frac{q}{2};\Omega},\norm{\bar{\sigma}}_{q-1;T}\le\mu_1+1$$ となることに注意する。

$\beta\ge 1$、$M\gt k$ とし、(\ref{E3})を $$G(t)=\begin{cases} t^\beta-k^\beta&\jf k\le t\le M\\ \beta M^{\beta-1}(t-M)+M^\beta-k^\beta&\jf t\gt M \end{cases}$$ として用いる。 $G'(t)=\begin{cases} \beta t^{\beta-1}&\jf k\le t\le M\\ \beta M^{\beta-1}&\jf t\gt M\\ \end{cases}, H(t)=\begin{cases} \frac{\beta^2}{2\beta-1}(t^{2\beta-1}-k^{2\beta-1})&\jf t\le M\\ \beta^2 M^{2\beta-1}(t-M)+\frac{\beta^2}{2\beta-1}(M^{2\beta-1}-k^{2\beta-1})&\jf t\gt M \end{cases}$ より $$G'(t)t\le\beta(G(t)+k^\beta),H(t)t\le\frac{\beta^2}{2\beta-1}(G(t)+k^\beta)^2\le 2\beta(G(t)^2+k^{2\beta})$$ かつ $$\frac{H(t)}{G'(t)}=\begin{cases} \frac{\beta}{2\beta-1}(t^\beta-k^{2\beta-1}t^{-(\beta-1)})\le G(t)&\jf k\le t\le M\\ \beta M^{2\beta-1}(t-M)+\frac{\beta}{2\beta-1}(M^{2\beta-1}-k^{2\beta-1}M^{-(\beta-1)})\le G(t)&\jf t\gt M \end{cases}$$ となることに注意すると $$\int_{\{u\gt k\}}|D(G(u))|^2\le C\left(\int_{\{u\gt k\}}\left(\beta^2\bar{b}+\bar{c}+\beta\bar{d}\right)G(u)^2+\int_{T\cap\{u\gt k\}}\beta\bar{\sigma}G(u)^2\right)+C\left(\int_{\{u\gt k\}}(\beta^2\bar{b}+\beta\bar{d})+\int_{T\cap\{u\gt k\}}\beta\bar{\sigma}d\H^{n-1}\right)k^{2\beta}.$$ (\ref{E1})、(\ref{E2})より $$\int_{\{u\gt k\}}|D(G(u))|^2\le \frac{1}{2}\int_{\{u\gt k\}}|D(G(u))|^2+C(\beta^\theta+1)\int_{\{u\gt k\}}G(u)^2+C\beta^2k^{2\beta},\theta=\theta_{n,q}\gt 2$$ となり $$\int_{\{u\gt k\}}|D(G(u))|^2\le C\beta^\theta\left(\int_{\{u\gt k\}}G(u)^2+k^{2\beta}\right).$$ Sobolevの不等式より $$\left(\int_{\{u\gt k\}}(u^\beta-k^\beta)^{2\chi}\right)^\frac{1}{\chi}\le C\beta^\theta\left(\int_{\{u\gt k\}}(u^\beta-k^\beta)^2+k^{2\beta}\right),\begin{cases} \chi\colon=\frac{n}{n-2}&\jf n\ge 3\\ \chi\gt 1&\jf n=2 \end{cases}.$$ 従って $u^+\in L^{2\chi\beta}(\Omega)$ で $$\norm{u^+}_{2\chi\beta}+k\le \left(\int_{\{u\gt k\}}(u^\beta-k^\beta)^{2\chi}\right)^\frac{1}{2\chi\beta}+Ck\le C^\frac{1}{2\beta}\beta^\frac{\theta}{2\beta}\left(\norm{u^+}_{2\beta}+k\right).$$ ここで $\beta=\chi^m\ (m=0,1,\ldots)$ ととると帰納法により $u^+\in L^{2\chi^m}(\Omega)\ (m=0,1,\ldots)$。さらに $$\norm{u^+}_{2\chi^{m}}+k\le\left(\prod_{l=0}^{m-1}C^{\chi^{-l}}\chi^{2l\theta\chi^{-l}}\right)\left(\norm{u^+}_2+k\right).$$ $m\to\infty$ として $$\sup_{\Omega}u^+\le C(\norm{u^+}_2+k)$$ を得る。$k$ のとりかたより主張が従う。

補題 63 (凸関数と劣解の合成)

$|\Omega|\lt\infty$ とする。$b_i,c_i,d,\sigma,f,g,\varphi_2$ は(\ref{C1})と(\ref{C2})と $$\int_\Omega\left(dv-\sum_i b_iD_iv\right)+\int_T \sigma vd\H^{n-1}\le 0\ (\forall v\in C^1_c(\OmT),v\ge 0) \tag{C5}\label{C5}$$ をみたすとし、$k\ge 0$ とする。$u\in W^{1,2}(\Omega)$ が $\displaystyle Lu\ge\sum_i D_if_i+g$ in $\Omega$、$\displaystyle Nu\le\sum_i\nu_if_i+\varphi_2$ on $T$、$w\colon=(u-k)^+$ をみたすとする。$H\in C^2([0,\infty))\cap C^{1,1}([0,\infty))$ は凸かつ非減少で $H(0)=0$ をみたすとすると $H(w)$ は $$\left\{\begin{align*} \sum_iD_i\left(\sum_ja_{ij}D_j(H(w))\right)+\sum_i(b_i+c_i)D_i(H(w))&\ge\sum_iH'(w)f_i-\left(\frac{1}{4\lambda}H^{\prime\prime}(w)|f|^2+H'(w)g\right)& &\inn\Omega\\ H(u)&=0& &\on\pOm\backslash T\\ \sum_{i,j}a_{ij}\nu_iD_j(H(u))&\le \sum_iH'(w)f_i\nu_i+H'(w)\varphi_2& &\on T \end{align*}\right.$$ をみたす。すなわち $H(w)\in W^{1,2}_0(\OmT)$ で $$\int_\Omega\left(\sum_{i,j}a_{ij}D_i(H(w))D_jv-\sum_i(b_i+c_i)D_i(H(w))v\right)\le\int_\Omega\left(H'(w)\sum_i f_iD_iv+\left(\frac{1}{4\lambda}H^{\prime\prime}(w)|f|^2+H'(w)g\right)v\right)+\int_TH'(w)\varphi_2vd\H^{n-1}\ (\forall v\in W^{1,2}_0(\OmT),v\ge 0).$$

Proof.

$H\in C^{2,1}(\R)$ の場合を示す。$v\in C^1_c(\OmT)$、$v\ge 0$ について \begin{align*} &\quad\L(u,H'(w)v)\\ &=\int_\Omega\left(\sum_{i,j}a_{ij}H'(w)D_juD_iv-\sum_ic_iH'(w)D_iuv\right)+\int_\Omega\left(\sum_ib_iD_i(H'(w)uv)-dH'(w)uv\right)+\int_T\sigma H'(w)uvd\H^{n-1}+\int_\Omega\left(H^{\prime\prime}(w)\sum_{i,j}a_{ij}D_iuD_juv-\sum_ib_iH'(w)D_iuv\right)\\ &\ge\int_\Omega\left(\sum_{i,j}a_{ij}H'(w)D_jwD_iv-\sum_i(b_i+c_i)H'(w)D_iwv\right)+\lambda\int_\Omega H^{\prime\prime}(w)|Dw|^2v. \end{align*} 一方、 \begin{align*} &\quad\int_\Omega\left(\sum_i f_iD_i(H'(u^+)v)+gH'(u^+)v\right)+\int_T\varphi_2H'(u^+)v\\ &=\int_\Omega\left(H'(u^+)\sum_i f_iD_iv+H^{\prime\prime}(u^+)\sum_i f_iD_iuv+H'(u^+)gv\right)+\int_TH'(u^+)\varphi_2v\\ &\le\int_\Omega\left(H'(u^+)\sum_i f_iD_iv+\left(\frac{1}{4\lambda}H^{\prime\prime}(u)|f|^2+H'(u^+)g\right)v\right)+\int_TH'(u^+)\varphi_2v+\lambda\int_\Omega H^{\prime\prime}(u^+)|Du|^2v. \end{align*} これより主張が従う。

定義 64 ($\sup_{\pOm\backslash T}u$)

$u\in W^{1,2}(\Omega)$ について $$\sup_{\pOm\backslash T}u\colon=\inf\{k\colon u\le k\ \inn\ \pOm\backslash T\}$$ と定める。$\inf_{\pOm\backslash T}u$ も同様に定義する。

注意

$\displaystyle k\ge\sup_{\pOm\backslash T}u$ とすると $(u-k)^+\in W^{1,2}_0(\OmT)$ が成り立つ。実際、$\displaystyle k\gt\sup_{\pOm\backslash T}u$ とすると $\displaystyle l\in\left(k,\sup_\Omega u\right)$ であって $(u-l)^+\in W^{1,2}_0(\OmT)$ となるものが存在し、$(u-k)^+=((u-l)^+-(k-l))^+$ であるから $(u-k)^+\in W^{1,2}_0(\OmT)$ も成り立つ。$\displaystyle k\to \sup_{\pOm\backslash T}u+0$ により $\displaystyle k=\sup_{\pOm\backslash T}u$ の場合も従う。

定理 65 (弱解の大域有界性2)

$\Omega$ は $|\Omega|\lt\infty$ かつ $1\notin W^{1,2}_0(\OmT)$ をみたすとする。$b_i,c_i,d,\sigma,f_i,g,\varphi_2$ は(\ref{C1})、(\ref{C4})をみたし、さらに

をみたすとする。

$u\in W^{1,2}_0(\OmT)$ が $\displaystyle Lu\ge(\le,=)\sum_i D_if-g$ in $\Omega$、$Nu\le(\ge,=)\varphi_2$ on $T$ をみたすとすると $$\sup_{\Omega}u(-u,|u|)\le C_{q,\Omega,T,\mu}\left(\sup_{\pOm}u^+(u^-,|u|)+\lambda^{-1}\left(\norm{f}_{q;\Omega}+\norm{g}_{\frac{q}{2};\Omega}+\norm{\varphi_2}_{q-1;T}\right)\right).$$

Proof.

  1. より広く、$\displaystyle\frac{\Lambda(x)}{\lambda(x)}$ が有界の場合に一様楕円型ということもある。
  2. 合成積とFourier変換で定義されているものとは $(2\pi)^\frac{n}{2}$ 倍の違いがある。